Download as pdf or txt
Download as pdf or txt
You are on page 1of 125

MODULE 1:

HISTORY OF FORENSIC CHEMISTRY

1
What this module is about

This module aims to give students a fundamental understanding of forensic chemistry. We


focus on the chemistry relating to specific evidences namely blood and bloodstains. This module
demonstrates the application of relevant techniques to forensic casework.

This module discusses the following topics:

Lesson 1: The History of Forensic Chemistry


Lesson 2: Evidence
Lesson 3: Types of Witness
Lesson 4: The Practice of Forensic Chemistry

Read this module and see the wonders of forensic chemistry and their contribution to the
world of elucidation of crime.

What you are expected to learn


After going through this module you are expected to:

1. Discuss the history of forensic chemistry.


2. Describe the different types of evidences.
3. List the golden rules in the practice of forensic chemistry
4. Explain the differences between ordinary and expert witness

How to learn from this module

Here’s a simple guide for you in going about the module:

1. Read and follow instructions carefully.


2. Answer the pretest in order to determine how much you know about the lessons in this module.
3. Read each lesson and do activities that are provided for you.
4. Perform all activities diligently to help and guide you in understanding the topic.
5. Take self-tests after each lesson to determine how much you understood the topic.
6. Answer the posttest to measure how much you have gained from the lessons.

2
What to do before ( Pretest)

I. Multiple Choice

Direction: Encircle the letter(s) of the BEST ANSWER.

1. Hearsay evidence is inadmissible except on the following:


a. res gestae b. dying declaration c. common reputation d. all of these

2. A kind of evidence which seeks to establish a conclusion by inferences from the facts
proven.
a. circumstantial evidence b. indirect evidence c. hearsay evidence d. all of theses

3. It is a proof of allegation.
a. violence b. evidence c. mistakes d. wrongful acts

4. One of the duties of a forensic chemist is to respond to queries of all investigating units
and render opinions.
a. true b. false c. partly false d. partly true

5. Other terms of real evidence


a. authentic evidence b. autoptic evidence c. austetic evidence d. autistic evidence

6. Cells that are produced by the bone marrow and are necessary for proper clotting of blood.
a. erythrocytes b. red blood cells c. leukocytes d. blood platelets

7. A test for blood which determines whether the stains contain blood or another substance.
a. confirmatory test b. blood grouping test c. precipitin test d. preliminary test

8. A preliminary test for blood which gives intense blue color that is produced immediately as
a positive result.
a. Phenolphthalein Test b. Guaiacum Test c. Benzidine Test d. Leucomalachite Green Test

9. The standard test used to determine whether the stain/blood is of human or animal origin.
a. confirmatory test b. blood grouping test c. precipitin test d. preliminary test

10. Discovered the four blood groups namely Group O, Group A, Group B and Group AB.
a. Land Steiner b. Beinsten c. Albert Einstein d. Michael Faraday

11. A condition wherein males have no spermatozoa at all in their seminal fluid.

a. Aspermia b. Oligospermia c. hypogonadism d. epididymitis

12. The following are examinations for seminal stains or seminal fluid except:
a. Florence Test b. Barberio’s Test c. Acid Phosphatase Test d. Phenolphthalein Test

3
13. A whitish fluid of male reproductive tract consisting of spermatozoa suspended in
secretion of accessory glands.
a. Spermatozoa b. Epithelial Cells c. Semen d. Seminal Fluid
14. Where can we find a fresh semen?

a. Vaginal contents of the victim


b. Hair
c. Skin around the genitals
d. Underclothings

15. In the collection, preservation, packing and transit of semen stained specimen, which
statement is incorrect?
a. Seizure of wearing apparel must be done anytime.
b.. In packaging of wearing apparel there should be no friction between the apparel and the stain.
c. Specimen should not be rolled for transit.
d. Specimen must be thoroughly dried before packing.

16. The following are factors that affect the presence or amount of gunpowder residues except:
a.Type and caliber of the ammunition
b. Length of the barrel of the gun
c. Sunlight
d. Wind velocity and direction

17. A test to determine whether a person fired a gun or not with bare hands.
a. paraffin test b. diphenylamine test c. spectroscopic test benzidine test

18. A blackening of area around the bullet hole.


a. Singeing b. Smudging c. Tattooing d. Soothing

19. A black substance that is formed by combustion, rises in fine particles and adheres to the
side of the barrel conveying the smoke.
a. Rust b. Soot c. Nitrate d. Nitrite

20. Combines with excess oxygen in an explosive to achieve oxygen balance, to prevent the
formation of nitrous oxides (toxic fumes), and to lower the heat of the explosion.
a. Bases b. Antacid c. Combustibles d. Absorbents

21. Outer coating of hair that is composed of overlapping scales.


a. medulla b. cuticle c. cortex d. root

22. Scale flattened consisting of overlapping scales with narrow margins that resemble puzzle
pieces. Commonly found in human hairs and many animal hairs.
a. spinous scales b. imbricate scales c. coronal scale d. soft scale

23. Fine hair present on the body after birth and before puberty.

4
a. Lanugos b. Vellus hair c. Terminal hair d. Full grown hair

24. The smallest unit of a textile material that has a length many times greater than its
diameter.

a. yarn b. fabric c. textile d. fiber


25. A convenient way for analysts to compare the colors of fibers through spectral patterns.

a. microspectrophotometer
b. chromatographic separation
c. Infrared spectrophotometry
d. Photochemistry

II. Essay

Direction: Answer the following question on the space provided.

1. In your own view, what is the most important type of evidence? (Ex. Blood, semen, knife etc.).

Answer:

5
MODULE 1: HISTORY OF FORENSIC CHEMISTRY

Intended Learning Outcome


 Identified the different types of evidence.
 Explained how to solve a crime by using evidence.

LESSON 1: HISTORY OF FORENSIC CHEMISTRY

The history of forensic chemistry dates back to ancient times when early civilizations used various
techniques to detect and analyze substances. However, the formal recognition and development of
forensic chemistry as a distinct scientific discipline began in the 19th century. Here's an overview of
the key milestones and advancements in the history of forensic chemistry:
1. Early Developments:
• 16th-17th centuries: Paracelsus, a Swiss alchemist, made significant contributions to
toxicology by studying the effects of chemicals on the human body.
• 18th century: Mathieu Orfila, a Spanish physician, is considered the father of forensic
toxicology. He published "Traite des poisons" in 1814, which became the foundation
for modern toxicology.
2. 19th Century:
• 1823: Friedrich Accum, a German chemist, published "A Treatise on Adulterations of
Food and Culinary Poisons," which highlighted the use of chemical analysis to detect
food adulteration.
• 1836: James Marsh, a British chemist, developed the Marsh test, a method to detect
the presence of arsenic in human tissue. This test played a crucial role in convicting
murderers in poisoning cases.
• 1850: Jean Servais Stas, a Belgian chemist, developed a highly accurate method to
detect arsenic in human organs.
• Late 19th century: Advances in analytical chemistry techniques, such as spectroscopy,
microscopy, and chromatography, expanded the capabilities of forensic chemistry.
3. 20th Century:
• Early 1900s: The development of forensic toxicology gained momentum.
Toxicologists began using techniques like thin-layer chromatography (TLC) and gas
chromatography (GC) to identify and quantify drugs and poisons in biological
samples.
• 1920s: August Vollmer, a leading figure in forensic science, introduced scientific
techniques in criminal investigations, including chemical analysis.
• 1932: The Federal Bureau of Investigation (FBI) established its forensic laboratory,
which included a chemistry division.
• 1944: The identification of the ABO blood groups by Karl Landsteiner, an Austrian
biologist, revolutionized forensic serology and blood typing.
• Late 20th century: The emergence of new technologies, such as mass spectrometry
and nuclear magnetic resonance (NMR), further enhanced the capabilities of forensic
chemistry.

6
4. Modern Era:
• Late 20th century to present: Forensic chemistry has continued to evolve with
advancements in DNA analysis techniques like polymerase chain reaction (PCR) and
short tandem repeat (STR) analysis, which have revolutionized forensic science.
• Contemporary forensic chemistry involves the analysis of various types of evidence,
including drugs, fibers, paint, gunshot residues, explosives, and trace elements.
Forensic chemistry plays a vital role in criminal investigations by providing scientific analysis and
evidence to assist law enforcement agencies and the justice system in solving crimes and ensuring
the integrity of legal proceedings.

LESSON 2: EVIDENCE

In the field of forensic science, evidence can be broadly categorized into several general types. These
types of evidence are applicable across various forensic disciplines, including forensic chemistry.
Here are the general types of evidence:
1. Physical evidence: This refers to any tangible object or material that is relevant to a crime.
Physical evidence can include weapons, tools, clothing, personal belongings, fingerprints,
footprints, tire tracks, and other objects found at a crime scene.
2. Documentary evidence: This includes any type of written or printed material that is used as
evidence in a case. It can include documents, contracts, letters, diaries, computer records,
emails, text messages, or any other form of recorded information.
3. Biological evidence: Biological evidence consists of any organic material that contains DNA,
such as blood, semen, saliva, hair, skin cells, or body tissues. It can be crucial in establishing
the identity of individuals, connecting suspects to crime scenes, or determining relationships
between individuals.
4. Digital evidence: In the digital age, electronic devices and data play a significant role in
investigations. Digital evidence encompasses computer files, hard drives, memory cards,
smartphones, tablets, emails, social media posts, internet browsing history, and other
electronic records that can provide insights into a crime or the actions of individuals involved.
5. Circumstantial evidence: Circumstantial evidence is indirect evidence that implies certain
facts or events without directly proving them. It includes information, circumstances, or
behavior that can be inferred to establish a person's guilt or innocence. Examples of
circumstantial evidence can include witness testimony, timelines, alibis, motives, or patterns
of behavior.
6. Testimonial evidence: Testimonial evidence consists of statements or testimonies provided
by witnesses or individuals involved in a case. It can include eyewitness accounts, expert
opinions, confessions, or statements given under oath.
7. Trace evidence: Trace evidence refers to small or microscopic fragments found at a crime
scene, such as fibers, hairs, paint chips, glass fragments, soil, or gunshot residue. It can
provide valuable information about the origin, transfer, and potential connections between
individuals, objects, and locations.
It's important to note that different types of evidence can be interconnected and used in combination
to build a comprehensive case. The specific types of evidence relied upon in a given investigation will
depend on the nature of the crime, the available evidence, and the objectives of the investigation.

7
What you will do
Activity 1.1

Direction: Read the paragraph below and on the space provide, list the following ( a. direct
evidence and b. indirect evidence) found.

While a policeman was walking his beat, he heard a scream come from the house. He ran to
the house and almost immediately he saw a man coming out of the house holding a bloody knife. The
policeman placed the man under arrest and entered the house. There he found a woman slumped to
the floor in a pool of blood with a stab wound on the breast.

Answer:

a. direct evidence

b. indirect evidence

What you will do

Activity 1.2
Direction: Analyze the image below and on the space provided, explain who the murderer is. A
woman is found dead in the ladies bathroom. Based on the picture below can you find the murderer?
How?

8
Answer:

What you will do


Activity 1.3
Direction: By seeing the image below, you need to identify if its murder or suicide. Present reasons.

Answer:

9
LESSON 3: TYPES OF WITNESS

ORDINARY AND EXPERT WITNESS

WITNESS – one who testifies in court and has personal knowledge or experience of
something. A person, other than the suspect who is requested to give information concerning an
incident or person. He may be a victim, a complainant, an accuser, a source of information and
an observer of an occurrence.

A witness in court may be an ordinary or expert witness. An ordinary witness is one who
states facts and may not express his opinions or conclusion. He may testify to impression of common
experience such as the speed of a vehicle, whether a voice was that of a man, woman or child. Beyond
this is closely limited.

As ordinary witness, the Rules of Court requires that the person must have the following
Qualifications:
1. He must have the organ and power to perceive.
2. The perception gathered by his organ can be imparted to others.
3. He does not fall in any of the exceptions provided for by the law, Sec. 26, Rules 123,
Rules of court.

An expert witness is one who possesses a special skill be it in art, trade or science or one
who has special knowledge in matters not generally known to men of ordinary education or
experience. He is a person skilled in some art, trade or science to the extent that he possesses
information not within the common knowledge of man.

DIFFERENCES BETWEEN ORDINARY AND EXPERT WITNESSES

1. An ordinary witness can only state what his senses have perceived while an expert witness may
state what he has perceived and also gives his opinions, deductions or conclusions to his perceptions.
2. An ordinary witness may not be skilled on the line he is testifying while an expert witness must be
skilled in the art, science or trade he is testifying.
3. An ordinary witness cannot testify on things or facts he has not perceived except those provided
for by the law while an expert witness may testify on things which he has not seen by giving his
opinions, deductions or conclusions on the statement of facts.

What you will do


Activity 1.4

Direction: On the space provided, explain your answer.


Can someone be a witness even if he/or she was not present when the crime happened?
Answer:

10
PROBATIVE VALUE OF EXPERT TESTIMONY

Whether the court are, or are not bound by the testimony of an expert, depends upon the
nature of the subject of inquiry. If the subject comes within the general knowledge of the judge,
the latter will not feel bound by the conclusion of the expert as for example the question of the
genuineness of a handwriting as compared to a standard
in issue. When, however, the subject of inquiry is of such a nature that a layman can have no
knowledge thereof, as for example, the determination of parentage by blood test, the court must be
dependent to expert evidence. In weighing the testimony of an expert, all the circumstances of the
case must be taken into consideration, among them (a) the degree of learning of the witness (b)
the basis and logic of conclusion (c) the other proof of case.

PRACTICE OF FORENSIC CHEMISTRY

The work of a forensic chemist is divided into four stages namely;

1. Collection or reception of the specimen to be examined – It is most important that


whenever possible the chemist should personally collect all the specimens necessary
for the examination. Unless it is done, something essential to the elucidation of the
problem may be omitted and is some cases, questions regarding the collection and
transit of the specimen are raised during trial.

The following guiding principles must be observed in the practice of forensic


chemistry:

a. SUFFICIENCY OF SAMPLES – Police are usually inclined to be niggardly in taking


samples probably because they have an unqualified belief in the magic of such
analytical instrument as the microscope and spectrograph. This mistake should be
avoided.

b. STANDARD FOR COMPARISON – If the evidence in question is found in the presence


of a foreign substance, a sample of the foreign substance must be submitted for analysis.

c. MAINTENANCE OF INDIVIDUALITY – Each evidence must be collected and


preserved as a separate sample. There must be no mixing or intermingling of unknown
with known.

d. LABELLING AND SEALING – Evidence will have no value in court in spite of the good
report of the expert if the specimen cannot be identified and possibility of tampering
excluded.

2. The actual examination of the specimen – The first step in the examination of an
article is to scrutinize it carefully and write down in the laboratory notebook a complete
description of its external appearance including the manner in which it is secured and
particulars of the sealing

3. Communication of the results of examination – The results of the


examination conducted will be communicated to the requesting party in the form of
a written report which must include an enumeration of the articles received for
examination with detailed description of the packing, sealing and labeling, date of

11
receipt and from whom received, the purpose of examination, the findings and
conclusion. The finding should include a brief but sufficient record of all significant facts
noted during the examination.

4. Court Appearance – The written report of the chemist is usually supplemented at a


later date by oral evidence if the case is brought to court or fiscal’s office. In court
appearance the witness must be composed and as much as possible avoid being
irritated by upbraiding of the opposite counsel. As Brouarded said: “ If the law has
made you a witness, remain a man of science. You have no victim to avenge, no
guilty or innocent person to ruin or save. You must bear witness within the limits
of science.”

What you will do


Activity 1.5
Direction: On the space provided, explain the meaning of the paragraph.

If the law has made you a witness, remain a man of science. You have no victim to
avenge, no guilty or innocent person to ruin or save. You must bear witness within
the limits of science.

Answer:

LESSON 4: THE PRACTICE OF FORENSIC CHEMISTRY

SIX GOLDEN RULES IN THE PRACTICE OF FORENSIC CHEMISTRY

1. GO SLOWLY
Good work cannot be hurried, therefore take all the time necessary to make the case
complete, no matter how urgent it may appear or how pressing others may be of the result; it is
generally possible to adjourn a case if the work cannot be finished in time.
2. BE THOROUGH
Make a careful and minute examination of everything and do not be satisfied with a
qualitative analysis if the quantitative one be possible; it always pays to do too much rather than too
little and it is difficult to foresee what will or will not be required in court.
3. TAKE NOTES
Keep a full, neat and clear record of everything seen and done.
4. CONSULT OTHERS
Many cases will lead the expert into paths which he is not familiar, and when this happens he
should consult others who are most likely to know.
5. USE IMAGINATION

12
This is somewhat hazardous advice, since an expert with a vivid and uncontrollable
imagination is a most dangerous person, a disciplined imagination, however, which enables
inferences and deductions to be made from slender and incomplete premise is often very useful.
6. AVOID COMPLICATED THEORIES
The simplest explanation is usually the right one.In the investigation of crimes, whether crime
against person or property, or even crime against a state, physical evidence is one of the most
important factors that should be given special attention. The prosecuting fiscal may win or lose a case
on the physical evidence presented to him by the investigator. It is probably the most damaging
evidence which can break down the hardened criminal. Unlike testimonial evidence, physical
evidence will not tell a lie.

However, these evidences that are very valuable become lost as far as prosecutive value is
concerned. Some of the primary reasons that may contribute to its disaster are:
1. Improper packing of specimen
2. Failure to identify the specimen
3. Improper precautions used in transmitting the specimen
4. Improper preservation
5. Lack of precaution to prevent tampering of specimen

MODULE 1: HISTORY OF FORENSIC CHEMISTRY SUMMARY

• Forensic Chemistry is a branch of chemistry that deals with the application of chemical
principles in the solution of problems that arise in connection with the administration of
justice.It is not limited to purely chemical questions involved in legal proceedings but invades
other branches of forensic sciences notably legal medicine, ballistics, questioned
documents, dactyloscopy and photography
• Scientific crime detection as such may well be described as owing its birth to the St.
Valentine’s Day Massacre which occurred in Chicago on February 14, 1929
• In the Philippines the first public recognition of the value of science in the proper
administration of justice was made when the position of “ Medicos Titulares” was created
in the Philippines by Virtue of the Royal Decree No. 188 of Spain dated March 31, 1876.
• Evidence may be (a) direct; (b) indirect, which includes circumstantial evidence and (c)
hearsay. Direct evidence is simply which the senses perceived. Circumstantial evidence
is a kind of evidence which seeks to establish a conclusion by inferences from proved facts.
Hearsay evidence is a statement made by a witness on the authority of another and
not from his on personal knowledge or observation. Real of Autoptic Evidence is an
evidence which is addressed to the senses of the court. Testimonial evidence is a solemn
declaration made orally by a witness under oath in response to interrogation by a lawyer.
Experimental evidence are results of certain experiments to prove a certain matter of fact.
Documentary Evidence is any written evidence presented by an expert in court which is
relevant to the subject matter in dispute and not excluded by the Rules of Court.
• A witness in court may be an ordinary or expert witness. An ordinary witness is one who
states facts and may not express his opinions or conclusion. An expert witness is one who
possesses a special skill be it in art, trade or science or one who has special knowledge in
matters not generally known to men of ordinary education or experience.

13
• The work of a forensic chemist is divided into four stages namely; the Collection or reception
of the specimen to be examined, the actual examination of the specimen, communication
of the results of examination, and court appearance.

References
Canete, A. M., & Sangil, M. B. (2014). Laboratory Manual for Forensic Chemistry and Toxicology ( with
Lecture Guide). Philippines: Wiseman's Book Trading.

Kintz, P., Salomone, A., & Vincenti, M. (2015). Hair Analysis in Clinical and Forensic Toxicology. United
Kingdom: Elsevier Inc.
Saferstein, R. (2014). Criminalistics: An Introduction to Forensic Science 11th Edition. Philippines:
Pearson .
Sanico, F. (2015). Forensic Chemistry Worktext. Philippines: Mutya Publishing Inc.

14
MODULE 2:
BLOOD AND BLOODSTAINS

15
What this module is about

The purpose of this module is to give students a fundamental understanding of forensic


chemistry. We focus on the chemistry relating to specific evidences namely blood and bloodstains.
This module demonstrates the application of relevant techniques to forensic casework.

This module discusses the following topics:

Lesson 1: Composition of Blood


Lesson 2: Importance of the Study of Blood
Lesson 3: Problems in the Study of Blood
Lesson 4: The Chronological Tests for Blood
Lesson 5: The Blood Grouping Test for Blood

Read this module and see the wonders of forensic chemistry and their contribution to the
world of elucidation of crime.

What you are expected to learn

After going through this module you are expected to:

1. Discuss the history of forensic chemistry.


2. Describe the different types of evidences.
3. List the golden rules in the practice of forensic chemistry
4. Explain the differences between ordinary and expert witness
5. Show / illustrate how forensic chemistry is important in the elucidation of crime.
6. Explain the practice of a forensic chemist.
7. Explain the four chronological tests for blood and the principles behind it.
8. Analyze the blood groupings.

How to learn from this module


Here’s a simple guide for you in going about the module:

1. Read and follow instructions carefully.


2. Answer the pretest in order to determine how much you know about the lessons in this module.
3. Read each lesson and do activities that are provided for you.
4. Perform all activities diligently to help and guide you in understanding the topic.
5. Take self-tests after each lesson to determine how much you understood the topic.
6. Answer the posttest to measure how much you have gained from the lessons.

16
MODULE 2: BLOOD AND BLOODSTAINS

Intended Learning Outcome

 Know the different tests for blood.


 Analyze the blood group.

Introduction
The significance of blood and bloodstains as evidence in crimes of violence is very obvious
such that we need not to place emphasis on this. The test for the identification of blood is employed
as an important part of the routine investigation in the many case of violent death. The specimen
usually submitted is fresh blood or fluid blood, dried blood and clotted blood. Very often it is brought
to the laboratory in the form of dried red or brown stains on weapons, clothing or other objects. Blood
– has been called the circulating tissue of our body. It is referred to us highly complex mixture of cells,
enzymes, proteins and inorganic substances. It is the red fluid of the blood vessels. Blood is opaque.
On treatment with either water or other reagents becomes transparent and assumes lake color. It is
faintly alkaline. Normal pH is 7.35 to 7.45.

LESSON 1 COMPOSITION OF BLOOD

1. 45% Formed elements or the solid materials consisting chiefly of cells.

a. Red Blood Cells or Erythrocyte – contain hemoglobin and carry oxygen to various cells in the body.
Circular, biconcave discs or rounded edges.

b. White Blood Cells or Leukocytes – are masses of nucleated protoplasm. It defends the body from
invading microorganisms. Help fight infection.

c. Blood Platelets or Thrombocytes – cells that are produced by the bone marrow and are necessary
for proper clotting of blood. Normally responsible for the retraction of blood clot.

2. 55% Plasma – the fluid portion of blood where the cells are suspended. It is principally composed
of –
a. Water (90%)
b. Solid (10%) – largely protein in nature and consists of albumen, several globulins and fibrinogen.

Albumen- the most abundant protein in the blood. It binds with many drugs

Globulins – has an important role in the immune mechanism of the body. The globulins carry drugs
as well as sex and thyroid hormones, lipids and iron. Fibrinogen- The soluble precursor of fibrin,
which form blood clot.

17
PLASMA – the yellowish fluids of the blood in which numerous blood corpuscles are suspended. A
straw-yellow liquid formed when blood to which an oxalate has been added to prevent clotting is
allowed to stand.

SERUM – a straw-yellow liquid formed when clotted blood is allowed to stand for some time and the
blood contracts.

LESSON 2: IMPORTANCE OF THE STUDY OF BLOOD

1. As circumstantial or corroborative evidence against or in favor of the perpetrator.


2. As evidence in case of disputed marriage.
3. As evidence in the determination of the cause of death and the length of victim survived the attack.
4. As evidence in the determination of the direction of the case of victim or assailant.
5. As evidence in the determination of the approximate time the crime was committed.

LESSON 3: PROBLEMS IN THE STUDY OF BLOOD

1. Where blood has to be searched?


In the collection of bloodstains, usually the attention is directed to clothing and weapons. We should
also look for bloodstains under the fingernails, linings of the pocket, seams and the folds of garments
of the suspect, under the edges of the table etc.

2. Collection, preservation, packing and transportation of specimen suspected to contain blood.

Blood offers little resistance to decomposition. It undergoes a rapid change in its character with the
passage of time as process of clotting and drying commences almost immediately on exposure to air.
Sodium fluoride may be added to blood to preserve it for a week room temperature of infinitely in a
refrigerator. Between 40 C -50 C is the ideal preserving temperature for blood and other perishable
specimens. Collections of bloodstains should be done as soon as possible. Mere washing of garment/
clothing removes the blood.

3. Does the stain contain blood or another substance?


The examination of the specimen should determine if the stain is blood, if it animal or human blood,
what blood groups are present.

LESSON 4: THE CHRONOLOGICAL TESTS FOR BLOOD

1. Preliminary Test- determines whether the stains contain blood or another substance. It is used to
demonstrate the presence of blood. It determines whether visible stains do or do not contain blood.

2. Confirmatory – Test that possibly identify blood. Determines whether bloodstain really contains
blood

18
3. Precipitin – determines whether the stain is of human or animal origin. Determines whether the
blood is of human or non-human origin, and if non-human, the specific animal family from which is
originated.

4. Blood Grouping Test – Determines the blood group if human blood.

I. Preliminary Test for Blood

A. The Benzidine Test

An extremely sensitive test that can be applied to minute stain. For many years the most common
used preliminary test for blood. Its used has generally been discontinued, as it is known carcinogen.
A very delicate test and will detect blood when present in 1:300,000 parts. The benzidine tests never
fails to detect blood even when very old, decomposed stain with all sorts of contamination is
examined. This test is more sensitive than guaiacum tests and is valuable as a negative result. If the
stain reacts negatively, it is not blood. The positive result is an indicative that blood may be present.

Reagent a.

Benzidine Solution ( a small amount of powdered benzidine dissolve din glacial acetic acid) b. 3%
solution of hydrogen peroxide

Procedure: Place a small fragment / portion of stained material on a filter paper. Add a drop of
benzidine solution and then a drop of hydrogen peroxide solution.

Positive Result: Intense blue color produced immediately.

Limitation of the test: Benzidine test is not specific test for blood. Positive result may be obtained
from the substances such as sputum, pus, natal secretion, plant juices, formalin, clay and gum. The
reaction is weaker and produce faint color

B. The Phenolphthalien Test

An alternative test to benzidine test. It can detect blood in a dilution of 80,000,000 parts. A positive
result with this test is highly indicative of blood. The negative result is therefore valuable and is
conclusive as to the absence of blood.

Reagent: a. Phenolphthalein solution ( 1 to 2 grams of phenolphthalein to 100 ml of 25% potassium


hydroxide in water added with one gram of zinc powder heated until colorless). b. 3% solution of
hydrogen peroxide

Procedure: Place a small fragment /portion of the stained material on a filter paper. Add a drop of
phenolphthalein solution and then a drop of hydrogen peroxide solution.

Positive Result: Rose color develops/deep pink/ permanganate color

Limitation of the Test: the test is also given by copper salts, potatoes and horseradish.

19
C. The Guaiacum Test

A fairly delicate test showing the presence of fresh blood in a solution 1:50,000 dilution. It
may not react to old stain.

Reagent: a. Fresh tincture of guaiacum resin ( few lumps of this to 95% alcohol, the filter) b. 3%
hydrogen peroxide solution for a few drops of turpentine.

Procedure: Place a small piece of the stained fabric on porcelain dish. Soak with fresh tincture of
guaiac. Add a few drops of hydrogen peroxide.

Positive Result: Beautiful blue color that appears immediately.


Limitation of the Test: The test also reacts with saliva, pus bile, milk, rust, iron salts, cheese, gluten,
potatoes, perspirations and other oxidizing substances.

D. The Leucomalachite Green Test

This test is not as sensitive as the benzidine test

Reagent: a. Leucomalachite green solution ( 1 gram leucomalachite green dissolved in 48 ml glacial


acetic acid and diluted to 250 ml water) b. 3% hydrogen peroxide

Procedure: Place a small piece of stained fabric on a filter paper. Add a drop of leucomalachite green
solution and after a few seconds add a drop of hydrogen peroxide.

Positive Result: Malachite green or bluish green

E. Luminol Test

An important presumptive identification for blood. The reaction of luminal with blood results
in the production of light rather than color. By spraying luminal reagent onto a suspected item, large
areas can be quickly screened for the presence of bloodstains. The sprayed object must be located in
a darkened area while being viewed for the emission of light. Luminol test is extremely sensitive test.
It is capable of detecting bloodstains diluted up to 10,000 times. Luminol is known to destroy many
important blood factors necessary for the forensic characterization of blood, so its use should be
limited only to seeking out invisible to the naked eye.

Positive Result: Luminescence or emission of light.

PRINCIPLE INVOLVED IN THE FOUR PRELIMINARY TESTS FOR BLOOD

The peroxidase present in hemoglobin acts as a career of oxygen from the hydrogen peroxide
to the active ingredients of the reagents ( benzidine, guaiac, phenolphthalein and leucomalachite) and
produces the characteristic colored compounds by oxidation. Peroxidase – an enzyme that
accelerates the oxidation of several classes of organic compound.

20
II. THE CONFIRMATORY TESTS FOR BLOOD

The actual proof that a stain is blood consists of establishing the presence of the characteristic
blood pigment hemoglobin or one of its derivatives. Hemoglobin is the red coloring mater of the red
blood cells of the blood.

THE THREE CONFIRMATORY TESTS FOR BLOOD

The three confirmatory tests for blood that determine whether stain is really blood are: a. Microscopic
Test b. Microchemical Test or Microcrystalline Test c. Spectroscopic Test

A. THE MICROSCOPIC TEST FOR BLOOD


The microscopic test is useful for the demonstration and mensuration of blood corpuscles
for making the distinction between mammalian, avian, piscine, and reptilian blood for the
investigation of menstrual, lochial and nasal charges. In short it differentiates mammalian, avian,
piscine and reptilian blood.

Method of Microscopic Examination:

1. Take two small fragments of the dry blood.


2. Place each fragment on separate slides with a drop of 0.9% salt solution.
3. The slides are put in a covered dish to prevent evaporation and the preparation allowed to stand
for 1-2 hours.
4. One of the slides is examined as wet preparation.
5. The other preparation is spread evenly over the slide, allowed to dry and stained by

a. Fix preparation in absolute methyl alcohol for 3 minutes. Stain in a 0.5% aqueous solution of eosin
for 1-3 minutes. Loffer’s methylene blue is added for 1-2 minutes. Eosin stains the red blood cells,
while methylene blue stains the nuclei.
b. Fix smear with methyl or ethyl alcohol for 3 minutes. Pour off alcohol and flood smear with
Geimsa’s stain. Stain for 15 minutes, cover to prevent evaporation, wash in water and dry.
c. Wright’s Stain – the smear is flooded with the stain and allowed to stand for a minute. Distilled
water is added until a metallic scum forms on the surface. Let stand for 3 minutes, circular wash with
water and dry.
Visible Results:
1. Mammalian Red Blood cells - circular, biconcave discs with nucleus. Appear as characteristics non-
nucleated discs. Exception is camel and closely related animal as llama whose red blood cells are oval
but also without nucleus.
2. Birds, fish and reptile red blood cells- large, oval and nucleated.
3. Amphibian red blood cells – are larger than mammals, oval and nucleated.
4. Lamprey eel red blood cells – circular and nucleated.

B. THE MICROCHEMICAL TESTS AND MICRPCRSYTALLINE TESTS FOR BLOOD

The identification of blood can be made more specific if microchemical or microcrystalline


test is applied or performed. Takayama test and Teicmann test are the two most popular ones. The
three Microchemical and Microcrystalline Test for Blood 1. Teicmann Haemin Reaction or Teicmann
Test or Haemin Crystal Test 2. Acetone-Haemin Test 3. Haemochromogen Crystal Test or Takayama
Test

21
a. THE TEICMANN TEST
The test depend on the addition of the specific chemicals to the blood so that characteristic
crystals with hemoglobin will be formed.

Reagent : Sodium Chloride, glacial acetic acid

Procedure: Place a minute fragment of the stain on the glass slide. Add a small crystal of sodium
chloride and 2-3 drops of acetic acid. Place cover slip and heat gently over a small flame to evaporate
the acid. Cool. Examine under the high power.

Postive Result: Dark brown rhombic crystal of haemin or haematin chloride arranged singly or in
cluster.

Limitation of the Test: the test is also given by indigo-dyed fabrics. If the stain is old or washed or is
changed by chemical reagetns, the crystals are not formed. The addition of too much salt or presence
of moisture in the acid or over-heating of the slide may result in failure.

b. THE ACETONE HAEMIN TEST

The test depends on the addition of specific chemicals to the blood so that characteristic
crystal with hemoglobin will be formed.

Reagent: Acetone, dilute acetic acid or oxalic acid

Procedure: Place dried stain on glass slide and cover with cover slip with a needle interposed to
prevent direct contact of the cover slip with the slide, add a drop of acetone and a drop of acetic acid.

Positive result: Small dark, diachronic acicular crsytals of acetone haemin

c. THE HAEMOCHROMOGEN CRSYTAL TEST OR THE TAKAYAMA TEST

A delicate test for the presence of hemoglobin. The test depends on the addition of specific
chemicals to the blood so that characteristic crystals of hemoglobin derivatives will be formed.

Reagent: Takayama Reagent ( 3 ml of 10% sodium hydroxide, 33 ml of pyridine, 3 cc saturated glucose


solution and diluted with 7 ml water)

Procedure: Place a small piece of suspected material on glass slide. Add a drop of Takayama Reagent.
Cover with a glass slip

Positive Result: Large rhombic crsytals of salmon pink color arranged in cluster heaves and other
forms that appear within 1 to 6 minutes when viewed under the low power objective. To hasten result
heat may be applied.

22
C. THE SPECTROCOPIC TEST FOR BLOOD

The most delicate and reliable test for the determination of the presence of blood both old
and recent stains. This test is performed by means of an optical instrument known as Spectroscope,
an optical instrument for forming and examining spectra.

Procedure: Dissolve bloodstain in water or saline solution. Place in small chamber (glass) with
parallel slides so arranged that the rays of light will pass directly through it. The chamber is placed in
the spectroscope and the instrument is so adjusted that the spectrum is clearly visible.

Positive Result: Upon absorbing some of the rays from the spectrum, it produced characetiristic dark
colored bands , which vary with the type of blood pigment.

Example: Oxyhemoglobin is marked by two bands


Hemoglobin – broad band

PRINCIPLE INVOVLED IN SPECTROCOPIC TEST

The absorption properties of translucent colored fluids can be observed on the solar
spectrum.

III. THE PRECIPITIN TEST FOR BLOOD

The precipitin test is the standard test used to determine whether the stain/blood is of human
or animal origin. The precipitin test is very sensitive and requires only a small amount of blood for
testing. Human bloodstain dried for as long as 10-15 years and longer may still a positive precipitin
reaction. Even extracts of tissues from mummies four to five years old have given positive reaction
with the test. Experience has shown that human bloodstain diluted by washing in water and left with
only a faint color may still yield a positive precipitin reaction.

Reagent: Precipitin/antiserum

Procedure: Scrape off bloodstain if on hard metal. Powder the scrapings and extract with saline
solution. If the stain is cloth, paper or similar material, cut a small portion and then place in a test
tube and add extract with saline solution. Allow the mixture to stand overnight. Centrifuge to clean
solution. Dilute with saline solution. Layer an extract of the bloodstain on top of the human
antiserum/precipitin in a capillary tube.

Positive Result:
1. Development of a white cloudy line at the contact point of fluids that appears immediately or within
1 to 2 minutes.
2. Human blood or for that matter, any protein of human origin in the extract will react specifically
with antibodies present in the serum as shown by the formation of cloudy ring or band at the interface
of the two liquids.

Principle involved in the Precipitin Test

When a rabbit is injected with human blood serum or whole human blood, the precipitin that
develops in its serum will react with the protein of the human blood serum, other human body fluids

23
and other human tissue extracts. The reaction is a specific one and if positive, will identify blood
proteins or any other protein as human origin.

Limitation of the Test: The precipitin reacts only with blood proteins but also with other body
proteins as those in saliva, semen, mucus and other body fluids. For this reason the test does not
identify specifically human body but only a protein material from the specific animal type. In order
that the conclusion of human blood is arrived, the precipitin test must be corroborated by
supplementary chemical, microscopic, or spectroscopic tests. The specificity and delicacy of the
precipitin reaction is great, but the reaction may be inhibited or may be destroyed by a number of
factors. Chemicals like acid, alkalis, alcohols, cresols, formaldehyde, corrosive sublimate or other
germicide may alter blood to such as extent that the reaction cannot be formed. Heat had the same
effect. Fluid blood loses its power of reacting with serum if its heated from 60 C to 90 C, while dried
blood may stand 150. Rust and postmortem decomposition may react with it poorly. Old stains may
be identified after a period of time.

What you will do

Self-test 2.1

Direction: Fill the blanks with the correct answers.


1. _________________ determines whether the stains contain blood or another substance.
2. __________________ Rose color develops/deep pink/ permanganate color as positive result.
3. ___________________ large, oval and nucleated.
4. ____________________ Luminescence or emission of light.
5. ____________________ Differentiates mammalian, avian, piscine and reptilian blood.

LESSON 5: THE BLOOD GROUPING TEST FOR FRESH BLOOD

If the specimen is human blood the next question is did it come from the victim, the accused
or from other persons? So the origin of blood or bloodstains will be determined by the identification
of the blood groups to which it belongs. This identification is carried out on both fresh blood and
bloodstains. Human blood of all races can be divided into definite groups. In the blood grouping of
fresh blood A-B-O system is used. It was Land Steiner who discovered the four blood groups namely
Group O, Group A, Group B and Group AB. He named the four groups on the basis of agglutinogen or
antigen content of the red blood cells. Antigens are characteristic chemical structures or “principles”
that are found on the surface of each red blood cell, which stimulates the profuction of agglutinins.
There are two agglutinogens classified as Agglutinogen A and Agglutinogen B on the other hand
serum contains proteins or principles known as antibodies or agglutinins, which cause agglutination
or clumping together of the red blood cells. There antitoxin substance within the body, which reacts
when confronted with a specific antigen to protect the system. There are two agglutinins classified as
Anti-A and Anti-B in the serum. Agglutinogen A and B are present at birth while agglutinins are
demonstrable in about 50% of newly born infants. If an individual belongs to group A this indicates
that his red blood cells has agglutinogen A located on its surface. Similarly all group B persons have
antigen B, All group AB persons have antigen A and B or group O persons have neither antigen A nor
Antigen B. When the serum of group A blood was examined, anti-B was found present and no Anti-A.

24
Similarly nGroup B contains only anti-A, Group O has both anti-A and anti-B and Group AB contains
either Anti-A or Anti-B.

Heredity of the Blood Groups Knowledge of the laws of genetic will make it easier to
understand the principle involved in the inheritance of blood groups. The inheritance of human blood
groups is pre determined by the presence or absence in the chromosomes of two factors or genes
called gene A and gene B. Since each body cell has a pair of chromosomes, each of which carries or
fails to carry on of these factors, an individual’s genetic constitution may be presented by AB, AA, AO,
BB, BO or OO where O represents the absence of chromosomes of either A or B factor. In the joining
of the ovum and spermatozoa during fertilization, a new pair of genes is formed corresponding to
the gene found in the chromosomes of the parents called zygote. If the genes are homozygous or pure
i.e. they are alike, and they are the same from both the father and mother, the characteristics are
transmitted unchanged from generation to generation. If the two genes are not the same which is
heterozygous or hybrid a new combination will arise in the next generation. Beinsten’s Theory of
Blood Group Inheritances The theory postulates that the presence of the three allelic genes A B and
O. According to him the blood group of nay individual is determined by combinations of A, B and O in
the particular pair of chromosomes. On gene is derived from the father the other gene from the
mother. Genes A and B are dominant over gene O. A and B determine the presence of corresponding
agglutinogens, while O determines their absence. The possible combination of this three genes
arranged in pairs give rise to six different genotypes corresponding to the four phenotypes or the
blood groups. There are then different mating possible between the four blood groups.

25
Grouping of Dried Bloostains The absorption technique or absorption-elution technique is an
indirect grouping technique of bloodstains and it depends on the detection of agglutinogen in the
dried blood. In dried blood grouping one cannot use the direct method as used in grouping fresh
blood because in direct groupings the identification of A and B antigens is accomplished by directly
reacting the blood with Anti-A and Anti-B serum. In dried blood, the red blood cells are already
ruptured due to drying leaving no cells in the stain to be agglutinated. However, although the cells
may have disintegrated, the antigens present on their surface remain and are still identifiable by
indirect means.

Importance of Blood Group Data Question of illegitimacy and relationship in many cases may
be solved by means of the blood groups.
1. Determination of whether a man accused of fathering a child out of wedlock could or not be its
parents.
2. Determination whether a child born of a married woman could or could not have been fathered by
her legal spouse.
3. Determination of whether a child could or could not belong to a given set of parents in the case of
accidental interchange of infants in hospital.
4. Determination of whether a child who has been lost and later recovered after a long interval could
or could not belong to a given set of parents.

26
What you will do
Self-test 2.2

Direction: Choose the letter(s) of the BEST ANSWER. Write your answers on the space provided.

1. In the ABO system, you normally can be


a. BO b. ABO c. A,B,AB,O d. all of these
2. Which of the following statement is true regarding the ABO blood system?
a. people who have antigen A normally would not produce the Anti A antibody
b. people who are type AB normally produce both Anti-A and Anti-B antibodies
c. The only ABO type blood that normally does not have either A or B antigens is AB.
3. The universal blood donors for the ABO system are type:
a. A b. B c. O d. AB
4. Which of the following statements is true about the ABO blood system?
a. It was discovered in 1950s
b. It was discovered by Karl Landsteiner
c. Few people are actually typed for this system because of the difficulty of procedure and high cost
d. A and B
5. If one of the parents is is blood type A and the other is Type B, which of the following blood
types would you likely be?
a. A b. B c. O d. AB

What you will do


Activity 2.1

Direction: Illustrate the chronological tests for blood by making a flowchart. Be sure to include the
principles behind the tests.

Module 2: Blood and Bloodstains Summary


• Blood has been called the circulating tissue of our body. It is referred to us highly
complex mixture of cells, enzymes, proteins and inorganic substances. Blood is
composed of 45% formed elements red blood cells or erythrocyte, white blood cells
or leukocytes and blood platelets or thrombocytes and 55% Plasma which is the fluid
portion of blood where the cells are suspended.

• There are four chronological tests for blood. Preliminary test determines whether
the stains contain blood or another substance. Confirmatory test identify blood. It
determines whether bloodstain really contains blood. Precipitin test determines
whether the stain is of human or animal origin. Blood Grouping test determines the
blood group if human blood.

27
• The Preliminary test for blood includes benzidine test, phenolphthalien test,
Guaiacum test, Leucomalachite Green test, and Luminol test .The confirmatory
tests for blood includes microscopic test, microchemical test or microcrystalline
test and spectroscopic test. In the blood grouping of fresh blood A-B-O system is
used. It was Land Steiner who discovered the four blood groups namely Group O,
Group A, Group B and Group AB.

• If an individual belongs to group A this indicates that his red blood cells has
agglutinogen A located on its surface. Similarly all group B persons have antigen B,
All group AB persons have antigen A and B or group O persons have neither
antigen A nor Antigen B.

References
Canete, A. M., & Sangil, M. B. (2014). Laboratory Manual for Forensic Chemistry and Toxicology ( with
Lecture Guide). Philippines: Wiseman's Book Trading.
Kintz, P., Salomone, A., & Vincenti, M. (2015). Hair Analysis in Clinical and Forensic Toxicology. United
Kingdom: Elsevier Inc.
Saferstein, R. (2014). Criminalistics: An Introduction to Forensic Science 11th Edition. Philippines:
Pearson .
Sanico, F. (2015). Forensic Chemistry Worktext. Philippines: Mutya Publishing Inc.

28
MODULE 3:
SEMEN AND SEMINAL STAIN

29
What this module is about

The purpose of this module is to give students a fundamental understanding of forensic chemistry.
We focus on the chemistry relating to specific evidences namely blood and bloodstains. This module
demonstrates the application of relevant techniques to forensic casework.

This module discusses the following topics:

Lesson 1: What is Semen?


Lesson2: Collection, preservation, packaging and transit of semen stained specimens
Lesson 3: The Examination of Semen and Seminal Stain
Read this module and see the wonders of forensic chemistry and their contribution to the
world of elucidation of crime.

What you are expected to learn

After going through this module you are expected to:

1. Discuss the history of forensic chemistry.


2. Describe the different types of evidences.
3. List the golden rules in the practice of forensic chemistry
4. Explain the differences between ordinary and expert witness
5. Show / illustrate how forensic chemistry is important in the elucidation of crime.
6. Explain the practice of a forensic chemist.
7. Explain the four chronological tests for blood and the principles behind it.
8. Analyze the blood groupings.

How to learn from this module

Here’s a simple guide for you in going about the module:

1. Read and follow instructions carefully.


2. Answer the pretest in order to determine how much you know about the lessons in this module.
3. Read each lesson and do activities that are provided for you.
4. Perform all activities diligently to help and guide you in understanding the topic.
5. Take self-tests after each lesson to determine how much you understood the topic.
6. Answer the posttest to measure how much you have gained from the lessons

30
MODULE 3: SEMEN AND SEMINAL STAINS

Intended Learning Outcome

 Know how to collect, preserve, pack and transit semen stained object.
 Analyze the existence of crime using the knowledge in forensic chemistry.

Introduction

The examination of semen and seminal stains is an important part in the crime investigation
of sexual offenses like cases of rape, adultery, sodomy, bestiality and sexual homicide.

Source: Center for Infectious Disease Research and Policy

Source: Labpedia.net

31
LESSON 1: WHAT IS SEMEN?

Semen- a whitish fluid of male reproductive tract consisting of spermatozoa suspended in secretion
of accessory glands.

Parts of the Semen


Seminal Fluid – has characteristic alkaline odor, it is viscid, gelatinous and sticky. Sometimes more
liquid in character when exposed to air for one and half hour due probably to enzymatic reaction.
Slightly alkaline in reaction.

Source: Trustyourperceptions.wordpress.com

Formed cellular elements which includes:


1. Spermatozoa or Sperm Cell – small objects with a pear-shaped head. Behind is a short neck and
then a tail of about ten times as long as the head.
2. Epithelial cells
3. Crystal of Choline and Lecithin

One point five 1.5 ml to 3.5 ml is the normal quantity of seminal fluid in single ejaculation.
400 to 500 million is the total number of spermatozoa contained in a single ejaculate from a healthy
young man.
Cases wherein ejaculation has no spermatozoa
1. Males suffering from Aspermia
Aspermia- a condition wherein males have no spermatozoa at all in their seminal fluid.

2. Males suffering from Oligospermia


Oligospermia- a condition wherby males have abnormally low sperm counts or with few
spermatozoa.

These two diseases result from excessive sexual intercourse, those suffering from chronic
epididymitis and ither testicular diseases. Also taken from chronic venereal diseases.

32
Where Semen can be found:
1. As Fresh
a. Vaginal contents of the victim
b. Rectal contents of the victim

2. As Wet or Dried Conditions


a. Hair
b. Skin around the genitals
3. As Dry Stains
a. Underclothing
b. Bed Clothing

LESSON 2: COLLECTION, PRESERVATION, PACKING AND TRANSIT OF


SEMEN STAINED SPECIMEN
1. Seizure of wearing apparel must be done as soon as possible. It often happened that washing the
clothes, chemise. Panties and trousers has destroyed important traces, skirts are the most common
parts of wear apparel carrying seminal stain.
2. In packaging of wearing apparel there should be no friction between the apparel and the stain. The
packaging of wearing apparel or objects carrying seminal stain must be made in such a manner that
there is no friction against the stain. Semen in dried condition is very brittle and is liable to break into
small particles which can be lost. Friction may cause the breaking of spermatozoa.
3. Specimen should not be rolled for transit. Gently lay between two sheets of cardboard or similar
material which are tied together with a string to avoid friction.
4. Smaller objects like hair should be placed in a test tube and corked.
5. Specimen must be thoroughly dried before packing. Presence of moisture certain bacteria act on
the protein constituents of semen, digest the dried protein and thus destroy its stiffness.
6. Fluid semen should be placed in a test tube. It may be preserved by a few drops of toulol or 10%
solution of formalin during hot weather there is a danger of putrefaction.

What you will do


Activity 1.1

Direction: Answer the following questions concisely.


1. Why should we thoroughly dry the semen before packing?

33
2. Can we pack the objects with seminal stain together with other evidences? Explain
your answer.

LESSON 3: THE EXAMINATION OF SEMEN AND SEMINAL STAIN

There are four examinations for seminal stains or seminal fluid namely:
A. Physical Examination
B. Chemical Examination
b.1 Florence Test
b.2 Barberio’s Test
b.3 Acid Phosphatase Test
C. Microscopic Examination
D. Biological Examination

A. The Physical Examination of Seminal Stain


a. Semen when dry gives stiff, starchy feeling to the cloth and produces slight deepening of the color
with the disappearance of odor. Stiffness disappears if semen is not properly dried in open air.
Presence of moisture bacteria will act on the protein constituent or semen, digest the dried protein
thus destroy its stiffness. The Bactria will remove the albuminous matter and also disintegrate the
spermatozoa.
b. Seminal stain exhibits bluish fluorescence under the ultra violet lights. Ultraviolet light is used to
locate invisible seminal stain or cloth. It gives bluish fluorescence provided the cloth is clean and not
dark colored. Bluish fluorescence is specific for seminal stains and may be seen in some other
albuminous materials.
c. Grayish white, sometimes yellowish stain which is typical of seminal fluid
d. Have appearance or outline of contour map
e. May have reddish tint in case of old man

B. The chemical Examination of Seminal Stains


1. Florence Test- Thus is known after the name of Dr. Florence of Lysons, who first introduced it.

Reagent/Chemicals:
Florence reagent (1.65 gram potassium iodide and 2,5 grams iodine in 30 cc of water)

34
Procedure:
1. Cut a portion of the stain and divide into small bits then soak in saline solution
2. Transfer into a slide, tease and evaporate the liquid.
3. Add a drop of Florence reagent cover with cover slip.
4. Examine under microscope.

Visible/Positive Result:
Crystals of choline perioiodide which are dark brown, rhombic or needle shaped that occur single or
in cross or even grouped in clusters. It resembles haemin crsytals in shape, size and color. Negative
reaction may be due to absence of seminal fluid or spermatic fluid may have not reacted with the
reagent due to the very low choline content because of over dilution. Florence test is only preliminary;
presence of spermatozoa confirms the presence of seminal stain.

Limitations of Florence Test


1. Clothes with seminal stains that are not dried thoroughlyso choline iodide is decomposed
completely so the result is negative.’
2. If stain is wet and mixed with blood.
3. After 24 hours it is negative due to rapid decomposition but still spermatozoa can be detected.
4. Even after a long period (2 ½ years) it will give positive result with Florence test provided
thoroughly dried and preserved and if free from blood and other albuminous substance.
If the seminal stain contains too much albumen as it is mixed with blood, the albumen interferes to
some extent in the test by reacting with so much of the iodine as to leave too little for the production
of Florence’s crystals.

2. Barberio’s Test
Reagent/Chemical:
Saturated aqueous or alcoholic solution of picric acid.
Procedure:
1. Soak a piece of stained material in a 2.5% solution of trichloroacetic acid for one hour in a test tube.
2. Centrifuge the test tube.
3. Get the clear liquid part and add to an equal amount of saturated aqueaous or alcoholic solution of
picric acid on a glass slide.
4. Observe under microscope

Positive Results
Crystal that are slender yellow tinted, rhomboid needles with obtuse angle or appear as ovoid
crystals. These crystals are made of specimen
Note: Barberio’s Test si almost specific for human semen. Seminal stain as old as six years are said to
respond to this test. This test is carried out with a fresh, dried or dissolved semen.
3. Acid Phosphatase Test.
This test is the best way to locate and at the same time characterized as seminal stain. It has
replaced the Florence test in reliability and was shown to be specific for human and higher apes. The
test is based fundamentally upon the extraordinary high acid phosphatase content of human male
ejaculate. Phosphatase is the enzymes present in semen.

35
Reagent: sodium a- naphtilphosphate and Fast Blue B. Dye
Procedure:
1. Moisten with water a piece of filter paper.
2. Swab the stained area with the filter paper.
3. The acid phosphatase will be transferred to the filter paper.
4. Add a drop of two or more of sodium alpha-naphtylphosphate and Fast Blue B Dry

Positive Result: Purple color.


Alternative Acid Phosphatase Test procedure:
Reagent: 23 grams of sodium chloride , 0.55 ml of glacial acetic acid, 2 grams of sodium
acectate trihydrate in 90 ml water , a suspension of 30 mg of anthraquinone-diazonium chloride and
50 grams of calcium-1-napthyl phosphate in 1 ml of 1% osol.
Procedure:
1.Treat the stained area in a water bath a pH 5 containing alpha napthyl phosphate as a substance
and anthraquinone-1-diazonium chloride.
2. Add the above ingredients.

Positive Result: Orange-red pigment


Principle of the test:
Alphanapthol by the acid phosphatase combines with the diazonium salt to the color. The
reaction takes place for 30 seconds on fresh stains.
Limitation of the test
Blood lengthens the time but does not interfere.
C. the Microscopic Examination of Semen and Seminal Stain
The chief purpose of microscopic examination is ti determine the presence of spermatozoa.
The identification of spermatozoa is at present the only specific test for the semen.
Procedure:
Determination of spermatozoa in fresh semen is relatively easier that its stains.
1. Transfer a drop of specimen to glass slide.
2. Add a drop of water or saline solution and cover with cover slip,
3. Examine under the high power objective
4. Observe for the presence of spermatozoa.

Determination of spermatozoa in seminal stain


1. A piece of material is teased on a slide in a drop of water/.
2. Allow the smear to dry and then stain with Loffler’s methylene blue for a minute , wash with water,
dry and examine under the microscope.
Limitation of the microscopic test
1. Absence of sperm does not prove that the stain have not been produced by human semen.

36
2. Elements which may obstruct detection of spermatozoa;
a. nature of fabric
b. age of stain
c. condition to which the stain was exposed before reaching the laboratory
d. handling the spectrum
3. Some medical jurist believes that there can be no semen without the presence of spermatozoa, but
not true in case of aspermia or oligospermia.

D. Biological Examination of semen and Seminal Stain


The spermato-precipitins are of value in the identification of seminal fluid in the certain cases
like for example: bestiality, when it may be desirable to differentiate between the human seminal
fluid from that an animal.
Farnum originally proposed this test in 1901. He injected human semen to a rabbit from five
to eight times of intervals from six to eight days. The serum obtained from the blood of the rabbit give
a precipitate with both recent and old emulsions of human semen. In 1928, Hektoem and Rustinant
showed that an antiserum produced by immunizing rabbits with human semen is both specie specific
and semen specific i.e. it gives a positive reaction to human blood.

Limitation of the test:


The bacterial action that produces disintegration of the spermatozoa in seminal stain is
equally effective in decomposing and digesting the protein constituents of semen that acts the antigen
producing antibodies. Such seminal stain with their protein constituent completely disintegrated
cannot possibly give precipitin reaction.

What you will do


Activity 3.1
Direction: A stained skirt was handed to you for an analysis. You run for the test in semen
analysis and found no spermatozoa in the microscopic test. In this case, do you think it is possible
that rape happened? Explain.

37
Other stains of medico-legal interest
1. Obstetrical and gynecological stains. Examination at the scene of crime in cases of criminal abortion
, infanticide, and sex offenses may lead to the discovery of dried linen, towels, chemise skirts,
matresses, blankets etc. which have stains.
2. Excrements:
Adults- yellowish brown
Infant- greenesh yellow
3. Paints stains: The criminal, in committing a crime may have brushed against a newly painted wall
or wall with loose water cement paint and may therefore carry some of the paint in his clothing.
4. Rust stains: Resembles bloodstains.
Rust- reddish-brown in color, insoluble in water and soluble in dilute acid.
5. Synthetic dyes: Resembles old bloodstains but can be recognized by treating strong acids and
alkaline
6. Mineral Acids: These are due to red paints containing oxides of iron.
7. Stains of vegetable origin: Stains resembling blood may be produced by fruit juices like mulberry
and mangosteen.
Almost all of the above can be differentiated from bloodstains by action of chemicals. The
above give reactions while blood does not.

Module 3: Semen and Seminal Stain Summary

• The examination of semen and seminal stains is an important part in the crime
investigation of sexual offenses like cases of rape, adultery, sodomy, bestiality and
sexual homicide.

• Seminal Fluid – has characteristic alkaline odor, it is viscid, gelatinous and sticky.
Sometimes more liquid in character when exposed to air for one and half hour due
probably to enzymatic reaction. Slightly alkaline in reaction.
• Semen can be found fresh in vaginal and rectal contents of the victim
• Semen can be wet or in dried conditions on hair and skin around the genitals.
• Semen can be found as dry stains on underclothing and bed clothing

• Seizure of wearing apparel must be done as soon as possible.


• In packaging of wearing apparel there should be no friction between the apparel and
the stain. 3. Specimen should not be rolled for transit.
• Smaller objects like hair should be placed in a test tube and corked.
• Specimen must be thoroughly dried before packing.
• Fluid semen should be placed in a test tube.

• There are four examinations for seminal stains or seminal fluid namely; physical
Examination, chemical examination, microscopic examination, and biological
examination.
• The chemical examination of seminal stains or seminal fluids includes Florence test,
Barberio’s test and Acid Phosphatase test.

38
References
Canete, A. M., & Sangil, M. B. (2014). Laboratory Manual for Forensic Chemistry and Toxicology ( with
Lecture Guide). Philippines: Wiseman's Book Trading.

Kintz, P., Salomone, A., & Vincenti, M. (2015). Hair Analysis in Clinical and Forensic Toxicology. United
Kingdom: Elsevier Inc.
Saferstein, R. (2014). Criminalistics: An Introduction to Forensic Science 11th Edition. Philippines:
Pearson .
Sanico, F. (2015). Forensic Chemistry Worktext. Philippines: Mutya Publishing Inc.

39
MODULE 4:
GUNPOWDER AND OTHER EXPLOSIVES

40
What this module is about

The purpose of this module is to give students a fundamental understanding of forensic


chemistry. We focus on the chemistry relating to specific evidences namely gunpowder and other
explosives. This module demonstrates the application of relevant techniques to forensic casework.

This module discusses the following topics:

Lesson 1: Gunpowder
Lesson2: Determination of whether or not a person fired a gun.
Lesson 3: Determination of the Possible Gunshot Range
Lesson 4: Determination of the probable time the gun has been fired
Lesson 5: Explosives
Read this module and see the wonders of forensic chemistry and their contribution to the
world of elucidation of crime.

What you are expected to learn

After going through this module you are expected to:

1. Discuss the history of forensic chemistry.


2. Describe the different types of evidences.
3. List the golden rules in the practice of forensic chemistry
4. Explain the differences between ordinary and expert witness
5. Show / illustrate how forensic chemistry is important in the elucidation of crime.
6. Explain the practice of a forensic chemist.
7. Explain the four chronological tests for blood and the principles behind it.
8. Analyze the blood groupings.

How to learn from this module

Here’s a simple guide for you in going about the module:

1. Read and follow instructions carefully.


2. Answer the pretest in order to determine how much you know about the lessons in this module.
3. Read each lesson and do activities that are provided for you.
4. Perform all activities diligently to help and guide you in understanding the topic.
5. Take self-tests after each lesson to determine how much you understood the topic.
6. Answer the posttest to measure how much you have gained from the lessons

41
MODULE 4: GUNPOWDER & OTHER EXPLOSIVES

Intended Learning Outcome

 Analyze gunshot range from the given evidence.


 Explain the principle behind explosion.

Introduction
In the investigation of crimes involving the use of firearms, three most important problems
may arise. The first and probably of primary importance is the problem of determining whether or
not a person has fired a gun with bare hands within a pertinent period of time. The other is the means
of determining the probable gunshot range i.e. the distance the firearm waS held from the body of the
victim at the time of discharged. A third problem may come up when the time of the firing of the gun
becomes an issue.

LESSON 1: GUNPOWDER

Two Kinds of Gunpowder:


1. Black powder – because of its inherent defects modern ammunition plants abandoned the use of
this.
2. Smokeless powder- is the most widely used propellant. It can either be a single base propellant or
double base propellant.

Source: CreativeCommon.org

Blackpowder – The oldest known explosive. It is consist of an intimate mixture of charcoal – 15%,
sulfur – 10% and potassium sodium nitrate – 75%. When exploded in open space the following
reaction occurs.
2KNO3 + 3C + S 3CO2 + K2S + N2

42
This reaction hold true if the composition of the powder is uniform, pure and no side reactions
take place. Slight difference in composition cannot be avoided as well as side reactions cannot be
controlled.
Smokeless Powder – the most widely used propellant. It is consists of cellulose nitrate or glyceryl
nitrate combined with cellulose nitrate and other stabilizers . Among the stabilizers used are nitrates,
bichromates and oxalates. Some of the organic stabilizers are nitrobenzene, graphite and Vaseline.
Stabilizers are added to reduce side reactions. These combine with the products of decomposition
and may have a negative or positive catalytic effect. When exploded the following reactions occur:
C12H14O4(NO3)6 9CO + 3N2 + 7H2) + 3CO2
Cellulose nitrate
4C3H3(NO3)3 12CO2 + 10 H2O + 6N2 + O2
Glyceryl nitrate

Possible locations of Nitrates when Black powder and smokeless powder explode
Nitrates are present in both gunpowder so that one will expect to find nitrates (NO) in the following.
1. Residue of the barrel of the gun
2. In or arounf the wound
3. On the clothing of the person fired upon at the close range
4. On the exposed surface of the hand of the person firing the gun

Factors that affect the presence or amount of gunpowder residues


1. Type and caliber of the ammunition – Different types of ammunition fired in the same weapon and
in the same distance may give different patterns
2. Length of the barrel of the gun- a weapon with 2 inches barrel will deposit residues over a larger
area than a weapon having a 5 inches barrel even though they are fired at the same distance and the
same ammunition.
3. Distance of the muzzle of the gun from a target.
4. Humidity – affects the speed with which the powder burns. Powder having lesser amount ot
moisture will burn more rapidly and completely within a given time yielding greater amount of
residue.
5. Wind velocity and direction – in high winds the residue will be blown in the direction of the winds
yielding a scattered pattern.
6.Direction of firing – firing vertically, slightly greater than firing horizontally from the same distance.
Powder residues have weight. When gun is fired downward or vertically all of the residues will fall on
the target, but when fired horizontally some of the residues are likely to fall short of the target.

43
Source: Compoundchem.com

44
LESSON 2: DETERMINATION OF WHETHER OR NOT A PERSON FIRED
A GUN WITH BARE HANDS

The burned residues are partially burned particles may escape around the breech of the gun
and implanted on the exposed surface of the hand firing the gun and the presence of these particles
serves as basis for the diphenylamine-paraffin test DPA-Paraffin Test.
Theory upon which the diphenylamine paraffin test is based
At the instance of discharge, there is a certain amount of gases, mixture, burned residues, and
partially burned particles that escape from the breech of the gun that may be implanted into the skin.
Diphenylamine paraffin test or dermal nitrate test or lunge
Diphenylamine test- a test to determine whether a person fired a gun or not with bare hands.
Procedure:
a. Paraffin Test – the taking of the cast to extract the nitrates embedded or implanted in the skin
b. diphenylamine test – the chemical aspect of the test. It determines the presence and distribution
of nitrates.

Reagent: Diphenylamine reagent ( 0.5 grams diphenylamine crystals dissolved in 100 ml of sulfuric
acid and 20 ml of water).
Visible Result:
Deep blue specks that develop when nitrates come in contact with the diphenylamine reagent.

Limitation of the Diphenylamine –Paraffin Test:

1. The test is not specific for nitrates since the role of nitrate is simply oxidizing agent. The test cannot
determine the source of nitrate.
2. There are other substances which contain nitrate oxidizing agents that are not in the ordinary
course of life like fertilizers, explosives, tobacco, firecrackers, urine, cosmetics and detergent.
3. Hands contaminated with nitrates from other sources than gunpowder or any oxidant one will
expect to find either a smear blue color or conglomeration of blue specks located at the different
places of the hand both dolsar and palmar aspects.

Possibilities that a Person may be Found Positive for Nitrates even if he did not actually fired
a gun.
1. It is possible that the gunpowder particles may have been blown on the hand directly from the
barrel of the gun being fired by another person.
2. An attempt to shield the body by raising the hand would in some instances result in the implanting
of powder particles on the hand of the person close to one firing the gun,

Possibilities that a Person may be Found Negative for Nitrates even if he actually fired a gun.
1. Use of automatic pistol

45
2. Direction of wind
3. Wind velocity
4. Excessive precipitation
5. Use of gloves
6. Knowledge of chemical that will remove the nitrates

In cases involving shooting incidents where paraffin test is required, the person
suspected to have fired a gun should be subjected to diphenylamine-paraffin test immediately and in
no case should it be postponed seventy-two (72 ) hours after shooting. It is possible to detect nitrates
as late as three days even though the hands have been washed. In the Philippines, the period is
reduced to 2 days only due to excessive perspiration.

LESSON 3. DETERMINATION OF POSSIBLE GUNSHOT RANGE

The clothing of the victim with bullet perforation should be submitted for possible gunshot
range.
How to collect, preserve and pack clothing.
Clothing removed from the victim should be cautiously and carefully handled to prevent
powder residues for becoming dislodged.
a. A large area as possible surrounding the gunshot hole should be made available for the test .
b. Do not wad the specimen or pack it loosely for shipment. Secure the area to be tested between two
layers of heavy cardboard fastened together tightly to prevent the specimen from becoming jostled
about in transit.
c. Each specimen should be wrapped separately.
d. Clothing heavily smeared with blood should be dried thoroughly before packing. If wet, they may
become mildewed or stick together in such a way that they will be unsuitable for the test.
e. The letter transmittal should contain all information as to existing circumstances and conditions
known to the investigator which may become helpful in making the test.

How to Determine the Probable Gunshot range


The clothing is examined microscopically for possible powder residue, singeing, burning,
smudging and powder tattooing.
Singeing – slight burning
Smudging – blackening of area around the bullet hole
Tattooing – Individual specks of nitrates around the bullet hole visible to the naked eye. It is black
coarsely peppered pattern.

Three zones of distances from which a firearm is discharged


1. Those in which the muzzle of the gun was held directly in contact with the body pr practically so
2. Those in which the muzzle of the gun was held 2 inches to 36 inches away.
3. Those in which the muzzle of the gun was held beyond 36 inches.

46
Held directly in contact: The characteristic patterns observed are as follows:
1. Gaping of hole where fabric is badly torn
2. Smudging
3. Singeing of the fibers at the entrance
4. Tattooing

Presence of partially burned powdered residues around the entrance hole that may be
embedded in the fabric. This could be present originally but may have become dislodged by rough
handling of the specimen or may have been blown into the wound or may have been washed by
bleeding.
Held from 2 inches to 8 inches (maximum)
The smoke and soot from the burned powder will be deposited around the hole of entrance
producing a dirty grimy appearance (covered with soot, dirt adhering or embedded on the surface).
Smudging around the perforation will be found to diminish in size as the muzzle of the gun is held a
distance of eight inches and all the blackening of the hole completely disappear and few individual
specks of tattooing will be visible with the naked eye.

Held from 8 inches to 36 inches


Tattooing is visible.
Held beyond 36 inches
Evidence of powder tattooing is seldom present.

47
What you will do
Activity 4.1

Direction: Analyze the image below and explain the gunshot range.

Answer:

48
LESSON 4: DETERMINATION OF THE PROBABLE TIME THE GUN HAS BEEN
FIRED

In the determination of the approximate time of last discharge, the specimen firearm is needed
in the examination.
At the crime laboratory, if the gun is examined immediately after the shooting the chemistry
rely more on the odor of the barrel. A characteristic smell will be present that decreases in intensity
with lapse of time, as smell of hydrogen sulfide. If the gun is examined later presence of nitrates,
nitrites, rust root and metallic fragments are determined.
Procedure: the barrel is swabbed with cotton with the aid of a barbecue stick and the presence of the
following is determined microscopically and chemically.
1. Soot – a black substance that is formed by combustion, rises in fine particles and adheres to the
side of the barrel conveying the smoke.
2. Metallic fragment
3. Rust- formation of rust inside the barrel after the gun has been fired is as good indication for the
determination of the approximate time the gun has been fired.
- If a gun has not been fired at all, no rust can be detected inside the barrel of the gun
-If a gun has been fired, iron salts are formed and are found inside the barrel. This iron salts is oxidized
resulting in the formation of the rust.
4. Nitrite – presence of nitrite (NO2-) is determined by addition of diphenyl reagent. If the color
becomes blue, nitrites are present and we may say that the firearm could have been fired recently.
5. Nitrates- presence of nitrates (NO3-) is determined by the addition of diphenyl reagent. If the color
becomes yellow green, nitrates are present and we may say that the firearm could have been fired
but not recently.

LESSON 5: EXPLOSIVES
Explosive Reactions

There are many chemical reactions that will release energy. These are known
as exothermic reactions. If the reaction proceeds slowly, the released energy will be dissipated and
there will be few noticeable effects other than an increase in temperature. On the other hand, if the
reaction proceeds very rapidly, then the energy will not be dissipated. Thus, a great quantity of
energy can be deposited into a relatively small volume, then manifest itself by a rapid expansion of
hot gases, which in turn can create a shock wave or propel fragments outwards at high speed.
Chemical explosions may be distinguished from other exothermic reactions by the extreme rapidity
of their reactions. In addition to the violent release of energy, chemical explosions must provide a
means to transfer the energy into mechanical work. This is accomplished by expanding product gases
from the reaction. If no gases are produced, then the energy will remain in the products as heat.
Most chemical explosions involve a limited set of simple reactions, all of which involve
oxidation (reaction with oxygen). A relatively easy way to balance chemical explosive equations is to

49
assume that the following partial reactions take place to their maximum extent (meaning one of the
reactants is totally consumed) and in order of precedence:

Theory of Explosives
In general, an explosive has four basic characteristics:
1. It is a chemical compound or mixture ignited by heat, shock, impact, friction, or a combination of
these conditions;
2. Upon ignition, it decomposes rapidly in a detonation;
3. There is a rapid release of heat and large quantities of high-pressure gases that expand rapidly with
sufficient force to overcome confining forces; and
4. The energy released by the detonation of explosives produces four basic effects; (a) rock
fragmentation; (b) rock displacement; (c) ground vibration; and (d) air blast.
The ingredients in explosives manufactured are classified as:
Explosive bases. An explosive base is a solid or a liquid which, upon application or heat or shock,
breaks down very rapidly into gaseous products, with an accompanying release of heat energy.
Nitroglycerine is an example.
Combustibles. A combustible combines with excess oxygen in an explosive to achieve oxygen
balance, to prevent the formation of nitrous oxides (toxic fumes), and to lower the heat of the
explosion.
Oxygen carriers. Oxygen carriers assure complete oxidation of the carbon in the explosive mixture,
which inhibits the formation of carbon monoxide. The oxygen carriers assist in preventing a lowering
of the exploding temperature. A lower heat of explosion means a lower energy output and thereby
less efficient blasting.
Antacids. Antacids are added to an explosive compound to increase its long term storage life, and to
reduce the acidic value of the explosive base, particularly nitroglycerin (NG).
Absorbents. Absorbents are used in dynamite to hold the explosive base from exudation, seepage,
and settlement to the bottom of the cartridge or container. Sawdust, rice hulls, nut shells, and wood
meal are often used as absorbents.
Antifreeze. Antifreeze is used to lower the freezing point of the explosive. Air gap sensitivity. Air gap
sensitivity is a measure of an explosive’s cartridge-to-cartridge sensitivity to detonation, under test
conditions, expressed as the distance through air at which a primed half-cartridge (donor) will
reliably detonate an unprimed half-cartridge (receptor).
Cap Sensitivity. Cap sensitivity is a measure of the minimum energy, pressure, or power required for
initiation of a detonation; i.e., “cannot be detonated by means of a No. 8 test blasting cap when
unconfined.
Strength Two strength ratings are used for commercial dynamites. Weight strength compares
products on an equal-weight basis, and cartridge strength or bulk strength compares products on an
equal-volume basis. Both are expressed in percent, using straight nitroglycerin dynamite as a
standard. A number of factors are currently used to judge an explosive's ability to do the work desired,
and today's blaster must consider at least the following:

50
Detonation Pressure is a measure of the product's shock wave energy, influenced by the product's
density (latent energy) and detonation velocity (rate of energy release).
Pressure Magnitude or Gas Pressure is a measure of the potential expanding-gas energy,
influenced by the product's density (latent gas volume) and the heat and velocity of detonation (rate
of gas production and expansion).
Forensic Investigation of Explosives
Explosives are substances that undergo a rapid oxidation reaction with the production of large
quantities of gases. It is this sudden buildup of gas pressure that constitutes the nature of an
explosion. The speed at which explosives decompose permits their classification as high or low
explosives. The most widely used explosives in the low-explosive group are black powder and
smokeless powder.
Among the high explosives, primary explosives are ultrasensitive to heat, shock, or friction and
provide the major ingredients found in blasting caps or primers used to detonate other
explosives.Secondary explosives are relatively insensitive to heat, shock, or friction and will normally
burn rather than detonate if ignited in small quantities in the open air.This group comprises the
majority of commercial and military blasting, such as dynamite, TNT, PETN, and RDX.
The Explosive Market
In recent years, nitroglycerin-based dynamite has all but disappeared from the industrial
explosive market and has been replaced by ammonium nitrate-based explosives (i.e., water gels,
emulsions, and ANFO explosives). In many countries outside the United States, the accessibility of
military high explosives to terrorist organizations makes them very common constituents of
homemade bombs. RDX is the most popular and powerful of the military explosives, often
encountered in the form of pliable plastic known as C-4.
Collection and Analysis
1. The entire bomb site must be systematically searched with great care given to recovering any
trace of a detonating mechanism or any other item foreign to the explosion site.
2. Objects located at or near the origin of the explosion must be collected for laboratory
examination.
3. Often a crater is located at the origin and loose soil and other debris must be preserved from
its interior for laboratory analysis.
4. One approach for screening objects for the presence of explosive residues in the field or
laboratory is the ion mobility spectrometer (IMS).
5. Preliminary identification of an explosive residue using the IMS can be made by noting the
time it takes the explosive to move through a tube. A confirmatory test must follow.
6. All materials collected for the examination by the laboratory must be placed in sealed air-
tight containers and labeled with all pertinent information.
7. Debris and articles collected from different areas are to be packaged in separate air-tight
containers.It has been demonstrated that some explosives can diffuse through plastic and
contaminate nearby containers.
8. Typically, in the laboratory, debris collected at explosion scenes will be examined
microscopically for unconsumed explosive particles.
9. Recovered debris may also be thoroughly rinsed with organic solvents and analyzed by
testing procedures that include color spot tests, thin-layer chromatography, high-
performance liquid chromatography, and gas chromatography-mass spectrometry.

51
10. Confirmatory identification tests may be performed on unexploded materials by either
infrared spectrophotometry or X-ray diffraction.

Schematic diagram of an ion mobility spectrometer.


A sample is introduced into an ionization chamber, where bombardment with radioactive particles
emitted by an isotope of nickel converts the sample to ions. The ions move into a drift region where ion
separation occurs based on the speed of the ions as they move through an electric field.

What you will do


Activity 3.1
Direction: Answer the following questions concisely.

1. What is an oxidizing agent? Why is an oxidizing agent important to an explosion?

52
2. Why does black powder not explode unless ignited in a confined area?

Module 4: Gunpowder & Other Explosives Summary


o There are two kinds of gunpowder, the black powder and the smokeless
powder, which is the most widely, used propellant.
• Nitrates can be found in the residue of the barrel of the gun, in or around the gunshot
wound, on the clothing of the person fired upon at the close range and on the exposed
surface of the hand of the person firing the gun.

• The factors that affect the presence or amount of gunpowder residues are the
type and caliber of the ammunition, length of the barrel of the gun, distance of the
muzzle of the gun from a target. humidity, wind velocity and direction and direction
of firing.
• Diphenylamine test is the test that determines whether a person fired a gun or not
with bare hands.
• Clothing removed from the victim should be cautiously and carefully handled to
prevent powder residues for becoming dislodged.
• The three zones of distances from which a firearm is discharged are; those in
which the muzzle of the gun was held directly in contact with the body practically so,
those in which the muzzle of the gun was held 2 inches to 36 inches away and those
in which the muzzle of the gun was held beyond 36 inches.
• At the crime laboratory, if the gun is examined immediately after the shooting the
chemistry rely more on the odor of the barrel. A characteristic smell will be present
that decreases in intensity with lapse of time, as smell of hydrogen sulfide. If the gun
is examined later presence of nitrates, nitrites, rust root and metallic fragments
are determined.
• Explosives are substances that undergo a rapid oxidation reaction with the
production of large quantities of gases. It is this sudden buildup of gas pressure that
constitutes the nature of an explosion. The speed at which explosives decompose
permits their classification as high or low explosives. The most widely used
explosives in the low-explosive group are black powder and smokeless powder.

53
References
Canete, A. M., & Sangil, M. B. (2014). Laboratory Manual for Forensic Chemistry and Toxicology ( with
Lecture Guide). Philippines: Wiseman's Book Trading.

Kintz, P., Salomone, A., & Vincenti, M. (2015). Hair Analysis in Clinical and Forensic Toxicology. United
Kingdom: Elsevier Inc.
Saferstein, R. (2014). Criminalistics: An Introduction to Forensic Science 11th Edition. Philippines:
Pearson .
Sanico, F. (2015). Forensic Chemistry Worktext. Philippines: Mutya Publishing Inc.

54
MODULE 5:
HAIR AND FIBER

55
What this module is about

The purpose of this module is to give students a fundamental understanding of forensic


chemistry. We focus on the chemistry relating to specific evidences namely blood and bloodstains.
This module demonstrates the application of relevant techniques to forensic casework.

This module discusses the following topics:

Lesson 1: Types of cases in which fibers may be of value as evidence


Lesson2: Morphology of hair
Lesson 3: Collection and Preservation of Hair
Lesson 4: Fiver Evidence

Read this module and see the wonders of forensic chemistry and their contribution to the
world of elucidation of crime.

What you are expected to learn

After going through this module you are expected to:

1. Discuss the history of forensic chemistry.


2. Describe the different types of evidences.
3. List the golden rules in the practice of forensic chemistry
4. Explain the differences between ordinary and expert witness
5. Show / illustrate how forensic chemistry is important in the elucidation of crime.
6. Explain the practice of a forensic chemist.
7. Explain the four chronological tests for blood and the principles behind it.
8. Analyze the blood groupings.

How to learn from this module

Here’s a simple guide for you in going about the module:

1. Read and follow instructions carefully.


2. Answer the pretest in order to determine how much you know about the lessons in this module.

56
3. Read each lesson and do activities that are provided for you.
4. Perform all activities diligently to help and guide you in understanding the topic.
5. Take self-tests after each lesson to determine how much you understood the topic.
6. Answer the posttest to measure how much you have gained from the lessons.

MODULE 5 : HAIR AND FIBER

Intended Learning Outcome

 Identify the parts of a hair and describe how does it grows.


 Explain how hair and fivers are used as evidence.

Introduction
Many crimes involve direct physical contact between victim and suspect. Whenever such
contact occurs, there is usually an inadvertent transfer of microscopic evidence. This transfer may
include fibers. However, these types of evidence, which can be microscopic in form, may often be
overlooked by investigating officers because they are not easily observed. Fibers can be classified as
animal (e.g. hair and silk), vegetable (e.g. cotton and linen), mineral (e.g. asbestos), or synthetic (e.g.
polyester and nylon), and are often identified by sub-classification through laboratory examination.
Cases in which fibers are being used as associative evidence are typically valuable in investigations
in which the suspect(s) are not known to have been to the location where the evidence was collected.

LESSON 1: TYPES OF CASES IN WHICH FIBERS MAY BE OF VALUE AS


EVIDENCE

Assault and Homicide - These types of crimes usually involve personal contact of some sort.
Therefore, clothing fibers and hair may be interchanged between victim and suspect; that is, fibers
and hairs from victim's clothing may be found on suspect's clothing and vice versa. Weapons and
fingernail scrapings may also be important sources of fiber evidence. Bindings, such as rope, may also
leave distinct fibers if a person was tied up.

Rape - The nature of this crime can result in the cross transfer of fibers and hairs between clothing
of victim and suspect and such articles as blankets or carpeting. Weapons and fingernail scrapings
may also be sources of fiber evidence.

Burglary - Clothing fibers may be found at the point where the burglar crawled through a window or
other opening, or climbed over a fence.

Hit-and-run - Due to the forceful contact between victim and automobile, clothing fibers and hair
may be found adhering to the fenders, grill, side mirrors or parts of the undercarriage. Fabric
impression patterns may also be observed on surfaces with which the fabric came into contac

57
LESSON 2: MORPHOLOGY OF HAIR

➢ Hair is composed of the protein keratin, which is also the primary component of finger and
toe nails.
➢ Hair is produced from a structure called the hair follicle. Humans develop hair follicles
during fetal development, and no new follicles are produced after birth.
➢ Hair color is mostly the result of pigments, which are chemical compounds that reflect
certain wavelengths of visible light.
➢ Hair shape (round or oval) and texture (curly or straight) is influenced heavily by genes.
The physical appearance of hair can be affected by nutritional status and intentional
alteration (heat curling, perms, straightening, etc.).
➢ The body area (head, arm, leg, back, etc.) from which a hair originated can be determined by
the sample’s length, shape, size, color, and other physical characteristics.
➢ In order to test hair evidence for nuclear DNA, the root must be present. The hair may also
be tested using mitochondrial DNA whether or not the root is present.
➢ It is the shaft, which is composed of three layers—the cuticle, cortex, and medulla—that is
subjected to the most intense examination by the forensic scientist.

Hair Structure
Hair is composed of three principal parts:

58
The structure of hair has been compared to that of a pencil with the medulla being the lead,
the cortex being the wood and the cuticle being the paint on the outside.

Cuticle – outer coating composed of overlapping scales

The cuticle is the scale structure covering the exterior of the hair.
➢ The scales always point towards the tip of the hair.
➢ The scale pattern is useful in species identification.
➢ The cuticle varies in its scales, thickness, and whether or not it
contains pigment.

Coronal Scales – crown-like usually on very fine diameter. Usually found in hairs of small
rodents and bats, but rarely humans

Spinous Scales – petal-like which are usually triangular shaped and protrude from the hair
shaft. Found on the fur of seals, cats, and some other animals. They are never found on human
hairs.
Imbricate Scales – flattened consisting of overlapping scales with narrow margins that
resemble puzzle pieces. Commonly found in human hairs and many animal hairs.

59
Cortex

The cortex is the main body of the hair shaft.


➢ Its major forensic importance is the fact that it is embedded with the
pigment granules that impart hair with color.
➢ The color, shape, and distribution of these granules provide the
criminalist with important points of comparison among the hairs of
different individuals.
➢ The cortex varies in thickness, texture and color
➢ Distribution of the cortex is perhaps the most important component
in determining from which individual a human hair may have come.
➢ Microscopic examination can also reveal the condition and shape of
the root and tip.

Medulla

The medulla is a cellular column running through the center of the hair.

➢ The medullary index measures the diameter of the medulla relative


to the diameter of the hair shaft.
➢ For humans, the medulla generally occupies less than one-third the
diameter of the shaft, while for animals it is generally one-half or
greater.

➢ The medulla may be continuous, interrupted, fragmented, or absent.

➢ Medullae also have different shapes, depending on the race or species.

60
➢ People of African or European descent may have fragmented medullae or have no
medulla at all.
➢ People of Asian descent usually have continuous medullae.
➢ Most other animals have continuous or interrupted medullae.
➢ The medulla may vary in thickness, continuity and opacity.
➢ It may also be absent in some species.
Root

The root and other surrounding cells in the hair follicle provide the tools necessary
to produce hair and continue its growth.
When pulled from the head, some translucent tissue surrounding the hair’s shaft
near the root may be found. This is called a follicular tag.
By using DNA analysis on the follicular tag, the hair may be individualized.

Hair from different parts of the body

The structure of hair differs on different parts of the body.


➢ Hairs from the scalp have consistent diameters and uniform distribution of pigment.
➢ Pubic hairs typically have continuous medullae
➢ Facial and beard hairs have triangular cross-sections and eyebrow hair decreases in
diameter from the root to tip.

Hairs differ at different points of growth and development

➢ Lanugos - a coat of delicate, downy hairs, typically found on human fetuses prior to birth.
Babies born prior to full term may retain the lanugo hair after birth for a short period.
➢ Vellus hair – fine hair present on the body after birth and before puberty.
➢ Terminal hair – larger, coarser hair of the adult.

Human Hair Growth

 Hair grows about one centimeter per month. The growth of human hair occurs in
three developmental stages called anagen, catagen, and telogen. (ACT)

 The anagen phase is the initial growth phase during which the hair follicle actively
produces hair and hair grows.

61
 The catagen phase is a transition phase between the anagen and telogen phases of
hair growth; hair is at rest and not actively growing.
The telogen phase is when the follicle is dormant or resting. During the telogen, hair
routinely falls from the skin. Hair in the telogen phase is the most common type of hair sample
found in crime scenes because hairs in this phase fall out with little to no provocation.
Comparing Strands

➢ The comparison microscope is an indispensable tool for comparing the


morphological characteristics of hair.
➢ When comparing strands of human hair, the criminalist is particularly
interested in matching the color, length, and diameter.
➢ A careful microscopic examination of hair will reveal morphological features
that can distinguish human hair from the hair of animals.
➢ Scale structure, medullary index, and medullary shape are particularly
important in animal hair identification.
➢ Other important features for comparing human hair are: The presence or
absence of a medulla, the distribution, shape, and color intensity of the
pigment granules present in the cortex.
➢ The most common request is to determine whether or not hair recovered at
the crime scene compares to hair removed from the suspect.
➢ However, microscopic hair examinations tend to be subjective and highly
dependent on the skills and integrity of the analyst.

Hair and DNA

Recent major breakthroughs in DNA profiling have extended this technology to the
individualization of human hair.
The probability of detecting DNA in hair roots is more likely for hair being examined
in its anagen or early growth phase as opposed to its catagen (middle) or telogen
(final) phases.
Often, when hair is forcibly removed a follicular tag, a translucent piece of tissue
surrounding the hair’s shaft near the root may be present.
This has proven to be a rich source of nuclear DNA associated with hair.

What you will do


Activity 5.1
Direction: Do the following activities on the space provided.

1. Draw the parts of hair and label it.

62
2. Draw and explain the life cycle of hair.

LESSON 3. COLLECTION AND PRESERVATION OF HAIR

1. Recover all hair present.


2. Use the fingers or tweezers to pick up hair, place in paper bindles or coin envelopes which
should then be folded and sealed in larger envelopes. Label the outer sealed envelope.
3. If hair is attached, such as in dry blood, or caught in metal or a crack of glass, do not attempt
to remove it but rather leave hair intact on the object.
4. If the object is small, mark it, wrap it, and seal it in an envelope.
5. If the object is large, wrap the area containing the hair in paper to prevent loss of hairs during
shipment.
6. In rape cases, the victim's pubic region should be combed prior to collecting standards.
7. Obtain known hair samples from the victim, suspect, or any other possible sources for
comparison with unknown specimens.
8. The recommended method for collecting head hairs is to start by having the person from
whom they are being collected bend over a large sheet of clean paper, rubbing or massaging
their hands through the hair so that loose hair will fall out on the paper. More should then be
gathered by plucking them from representative areas all over the head. A total or 50-100
hairs is desired. Do not cut the hair. This same method may be used to collect hairs from other
parts of the body. 30-60 pubic hairs are required. When the person is a suspect, hair should
be gathered from all parts of the body even though there may only be an interest in hair from
the head at that particular time.
9. Hair samples are also collected from the victim of suspicious deaths during an autopsy.

63
What you will do
Activity 5.2
Direction: Answer the question on the space provided.
1. Aside from DNA testing, what are the other tests for hair and why are those tests
conducted?

Answer:

LESSON 4. FIBER EVIDENCE

Fiber Evidence
A fiber is the smallest unit of a textile material that has a length many times greater than its
diameter. A fiber can be spun with other fibers to form a yarn that can be woven or knitted to form
a fabric. The type and length of fiber used, the type of spinning method, and the type of fabric
construction all affect the transfer of fibers and the significance of fiber associations. This becomes
very important when there is a possibility of fiber transfer between a suspect and a victim during
the commission of a crime. Matching unique fibers on the clothing of a victim to fibers on a suspect’s
clothing can be very helpful to an investigation, whereas the matching of common fibers such as
white cotton or blue denim fibers would be less helpful. The discovery of cross transfers and
multiple fiber transfers between the suspect's clothing and the victim's clothing dramatically
increases the likelihood that these two individuals had physical contact.
Types of Fibers

A textile is an artifact made by weaving, felting, knitting, or crocheting natural or synthetic


fibers. Textiles can be initially categorized by their weave patterns. Further examination of the
individual fibers would reveal the type of fiber, length, color, and method of spinning. Fibers can also
be classified as natural or synthetic (man-made).

Natural fibers are derived in whole from animal or plant sources.


➢ Examples: Wool, mohair, cashmere, furs, and cotton.

64
Originating Animal Fiber Uses

Sheep Wool Clothing, carpeting, blankets, rugs.

Rabbit Angora Clothing (sweaters)

Goat Mohair and Cashmere Clothing

Silkworm Silk Clothing

Alpaca Alpaca Blankets, Clothing,

Muskox Qiviut Hats, scarves (expensive)

Originating Plant Fiber Uses

Cotton Cotton Most common plant fiber used. Clothing, Towels,


Blankets,
Flax Linen Clothing, towels, napkins, table cloths, formerly used
for Bed sheets,

Jute Jute 2nd most common plant fiber used. Burlap or Gunny
Cloth for sacks, animal feed and seeds, bags for the
roots of trees, some clothing
Coconut Tree Coir Doormats, brushes, sacks, twine, in horticulture

Cannabis Plant Hemp Food products, oil, rope, wax, cloth, paper, fuel

China Grass Ramie Binding for books, industrial sewing thread, fishing
nets (often blended with other fibers such as cotton)
Agave Plant Sisal Rope, twine, cloth, rugs, carpets, dart boards

Man-made fibers or synthetic fibers are manufactured.


➢ Regenerated fibers are manufactured from natural raw materials and
include rayon, acetate, and triacetate.
➢ Synthetic fibers are produced solely from synthetic chemicals and
include nylons, polyesters, and acrylics.
➢ Polymers, or macromolecules, are synthetic fibers composed of a
large number of atoms arranged in repeating units known as
monomers.

65
Man-made Fibers

➢ Mad-made fibers made from natural or synthetic polymers (composed for a large
number of atoms usually arranged in repeating units). Ex. Nylon, polyester, and
acrylics.
➢ Regenerated Fibers – are man-made from regenerated cellulose which comes from
wood or cotton pulp. Ex. Rayon, acetate, triacetate
➢ Polyester, most popular synthetic fiber
➢ Nylon, created in 1939 by Dupont, is the 2nd most popular synthetic fiber

Fibers as Physical Evidence

Fibers found at the crime scene often present challenges to investigator. Fibers can
sometimes be linked to a suspect if the fiber is sufficiently unique and is found in the suspect’s
possession and at the crime scene.

Locard’s Exchange Principle

Locard's exchange principle states that materials are exchanged when two objects come in
contact with each other. The exchange of fiber evidence can occur as the clothing of the victim and
the suspect come in contact with each other as well as fibers at the crime scene.

Identification and Comparison of Man-Made Fibers

 The combined factors of color, size, shape, microscopic appearance, chemical composition,
and dye content make it very unlikely to find two different people wearing identical fabrics.
 The physical characteristics of fibers may be examined by the following:
 Counting the number of filaments
 Calculating the density of the fiber
 Evaluating the Refraction Index of the fiber
 Checking the fiber for fluorescence
 The chemical characteristics of fibers may be examined in the following ways:
 Oxidation tests: This involves burning the fiber to evaluate the ash, behavior in the
flame and any smells associated with the fiber.
 pH Testing
 Evaluating any residues or component parts within synthetic fibers
 Chemical Decomposition tests: Treating fibers with strong acids, bases or solvents

Methods for Fiber Comparison

➢ The visible light microspectrophotometer is a convenient way for analysts to compare the
colors of fibers through spectral patterns.
➢ A more detailed analysis of the fiber’s dye composition can be obtained through a
chromatographic separation.
➢ Infrared spectrophotometry is a rapid and reliable method for identifying the generic class of
fibers, as does the polarizing microscope.
➢ Depending on the class of fiber, each polarized plane of light will have a characteristic index
of refraction.

66
Collection, Preservation and Marking of Fiber Evidence

Before attempting specific procedures listed below, note the following general precautions:
1. The size of the container should correspond to the size of the object.
2. Do not package wet evidence. Fibers or objects containing fiber evidence should be air dried
before placing in sealed containers. Biological stains degrade with time. This process is accelerated
when items are wet and sealed in airtight containers.
3 Do not package items on a surface without first thoroughly cleaning that surface. Avoiding cross
contamination between all evidence and standards is imperative!
4. All seams of the packaging must be sealed to prevent the loss of trace evidence. 5.
Label all evidence containers with submitter’s initials, ID/badge number, agency name, case number,
item number, source, and date.

Collection Procedures

1. Where fibers are visible and firmly attached to an inanimate object to be


transported to the laboratory:
Leave fibers intact.
(a) Diagram and note exact location and approximate number of fibers adhering
to each object (photograph if possible).
(b) Label object and package in a container so that fibers cannot become
dislodged in transit.
(c) Label packaging with appropriate information.
2. Where fibers are visible and not firmly attached, or if firmly attached and object
is too large to send to the lab:
(a) After diagramming and noting each location and the number of fibers
present, carefully remove with clean tweezers and package.
(b) Place fibers in a small pill box, glass vial or other tightly sealed container.
Fibers may also be placed in small folded paper bindles.

(c) Label packaging with appropriate information.


3. Where fibers are possibly transferred to clothing of victim or suspect:
(a) Be sure clothing is dry before packaging.
(b) Keep each item separate.
(c) Avoid disturbing soil, dust, blood, seminal stains, or other foreign materials adhering to clothing.
(d) If any of the aforementioned are apparent, see appropriate Physical Evidence Bulletin for
special instructions.
(e) Place ID mark on each item in an easily located area that does not damage the clothing.
(f) After allowing wet apparel to air dry, carefully fold and wrap each article separately, package, and
label with appropriate information (layers of clean wrapping paper and new paper bags are suitable
for this purpose).
4. For fingernail scrapings/clippings:
(a) Take scrapings/clippings from both suspect and victim.
(b) Use either a clean knife, clippers, or other instrument such as a
fingernail file or toothpick.
(c) Use a separate folded paper bindle for each hand to collect
scrapings/clippings.

67
(d) Place the folded and labeled bindles (i.e. “left hand”, “right hand”) in a pill box, glass vial or other
small tightly sealed container and label with appropriate information.
5. Where fibers are in hair of suspect or victim:
Comb the individual's hair over clean white paper using a clean fine-tooth comb. Carefully fold the
paper together with the comb into a bindle to prevent loss of any trace evidence. Place the bindle in
an envelope and label with appropriate information.
Collection of Fiber Standards for Comparison

FIBER STANDARDS: You need to have a fiber standard from an individual or the scene in
order to try and place a victim or suspect at a location. If a someone can already be placed at a specific
location the fiber analysis has limited value. When fibers have been collected by the investigating
team it is imperative that appropriate and adequate standard samples also be submitted. For
example, if fibers are found on the soles of the robbery suspect's shoes, standard samples of the
carpet or carpets at the crime scene should also be submitted. The standard samples should be a
representative sampling and include variations due to color, style, type, fading, staining, or wear.
Standard samples with a minimum size of a quarter should be submitted.
It will not always be known to the investigating officer whether there are fibers present in the
submitted evidence. For this reason, care must be exercised when handling any item that could shed
fibers and thereby cause cross contamination between items from suspects and victims.

Results Possible from Laboratory Examination of Fiber Evidence

Fibers
a. Fiber classification (i.e. animal, vegetable, mineral, or synthetic) and sub-classification (e.g.
polyester, nylon, acrylic).
b. Determination as to whether questioned fibers are the same type and similar color as the
standard. Determination as to whether questioned and standard fibers share similar microscopic
characteristics. (Note: Color and microscopic characteristics of fibers may vary within a garment,
carpet, drape, rope, etc. due to many factors, such as wear or fading.)
c. An opinion as to whether questioned fibers could have originated from the standard.

What you will do


Activity 5.3
Direction: Answer the question on the space provided.

1. Why is fiber evidence not considered as reliable evidence?

68
Answer:

Module 5: Hair and Fiber Summary


Fibers may be of value as evidence in cases such as assault and homicide, rape ,burglary and
hit-and-run.

Hair is produced from a structure called the hair follicle. It is the shaft, which is
composed of three layers—the cuticle, cortex, and medulla—that is subjected to the most
intense examination by the forensic scientist.

The comparison microscope is an indispensable tool for comparing the morphological


characteristics of hair. A careful microscopic examination of hair will reveal morphological features
that can distinguish human hair from the hair of animals.

Scale structure, medullary index, and medullary shape are particularly important in animal
hair identification.

A fiber is the smallest unit of a textile material that has a length many times greater than its
diameter. The type and length of fiber used, the type of spinning method, and the type of fabric
construction all affect the transfer of fibers and the significance of fiber associations. The discovery
of cross transfers and multiple fiber transfers between the suspect's clothing and the victim's
clothing dramatically increases the likelihood that these two individuals had physical contact.

Natural fibers are derived in whole from animal or plant sources.

Man-made fibers or synthetic fibers are manufactured.


Locard's exchange principle states that materials are exchanged when two objects come
in contact with each other. The exchange of fiber evidence can occur as the clothing of the victim
and the suspect come in contact with each other as well as fibers at the crime scene.

69
The visible light microspectrophotometer is a convenient way for analysts to compare the
colors of fibers through spectral patterns. A more detailed analysis of the fiber’s dye composition
can be obtained through a chromatographic separation. Infrared spectrophotometry is a rapid
and reliable method for identifying the generic class of fibers, as does the polarizing microscope.

References
Canete, A. M., & Sangil, M. B. (2014). Laboratory Manual for Forensic Chemistry and Toxicology ( with
Lecture Guide). Philippines: Wiseman's Book Trading.

Kintz, P., Salomone, A., & Vincenti, M. (2015). Hair Analysis in Clinical and Forensic Toxicology. United
Kingdom: Elsevier Inc.
Saferstein, R. (2014). Criminalistics: An Introduction to Forensic Science 11th Edition. Philippines:
Pearson .
Sanico, F. (2015). Forensic Chemistry Worktext. Philippines: Mutya Publishing Inc.

70
MODULE 6:
CHEMICAL ASPECT OF DOCUMENT
EXAMINATION

71
What this module is about

The purpose of this module is to give students a fundamental understanding of forensic


chemistry. We focus on the chemistry relating to specific evidence namely documents. This module
demonstrates the application of relevant techniques to forensic casework.

This module discusses the following topics:

Lesson 1: Document
Lesson 2: The Examination of Questioned Documents
Lesson 3: The Examination of Paper
Lesson 4: The Four Tests for Paper
Lesson 5: The Analysis for Ink

Read this module and see the wonders of forensic chemistry and their contribution to the
world of elucidation of crime.

What you are expected to learn


After going through this module you are expected to:

1. Differentiate original and fake documents.


2. Know how to analyze questioned documents.

How to learn from this module

Here’s a simple guide for you in going about the module:

1. Read and follow instructions carefully.


2. Answer the pretest in order to determine how much you know about the lessons in this module.
3. Read each lesson and do activities that are provided for you.
4. Perform all activities diligently to help and guide you in understanding the topic.
5. Take self-tests after each lesson to determine how much you understood the topic.
6. Answer the posttest to measure how much you have gained from the lessons.

72
MODULE 6: CHEMICAL ASPECT OF DOCUMENT EXAMINATION

Intended Learning Outcome

 Differentiate original and fake documents.


 Know how to analyze documents

Introduction
At first impression it seems that the examination of questioned documents is hardly within
the province of the forensic chemist, but if we consider the fact that the essential materials in a
document examination of any kind are the paper and ink or pencil, and the chemical examination of
inks, erasures, alterations and sequence or writing are often associated with such examination, it will
be very evident that there is a large amount of purely chemical work in document examination.

LESSON 1: DOCUMENT

Document – An original or official written or printed-paper furnishing information or used as a proof


of something else. Is any object that contains handwritten or typewritten markings whose source or
authenticity in doubt.
Packing, Preservation and Transportation of Documents
Documents are precious things and therefore should be treated accordingly.
1. Documents should be handled, folded, and marked as little as possible.
2. If folding is necessary to send to the laboratory, the fold should be made along old lines. Place
it in Manila paper envelope or brown envelope since it is sufficiently hard paper, or it can be
placed in a transparent plastic envelope.
3. On receipt the document should be placed between two sheets of plain white paper in a
folder.
4. Documents should be touched with pencil, pen or anything that could have possibly marked
them.

LESSON 2: THE EXAMINATION OF QUESTIONED DOCUMENTS

Examination and Comparison of Paper


The essential materials in a document examination of any kind are the paper and ink or pencil
or writings. The examination of paper may be necessary if we want to know the age of the document,
the presence of alterations, erasures, and other forms of forgery.

73
Problems Encountered in the Analysis of Paper
1. Determination of whether two pieces of paper originated from the same source.
2. Determination of the probable age of paper.
3. Determination of the composition of the paper.

Fiber Composition – practically all papers maybe classified from the standpoint of basic fiber
composition into sets of fiber mixtures namely:
a. Mechanical pulp- ground wood sulfite mixture, this is pulp from coniferous dicotyledonous
wood in combination with sulfite chemical pulp from conifers.
b. Soda-sulfite mixture – chemical pulp from conifers.
c. Rag sulfite – cotton rag or linen rag.

Sizing material – added to paper to improve its texture. Examples of sizing materials are rosin,
casein, gelatin, and starch.
Loading material- Added to paper to give weight. It partially fills the pores between the fibers of
the paper. Examples of loading materials are calcium sulfate and barium sulfate.

LESSON 3: THE EXAMINATION OF PAPER

The examination and comparison of paper may determine the following:


1. The age of the paper as compared to the age of known document.
2. Whether the paper is identical with or different from another paper whose history is
known.
3. Whether two sheets of paper of the same manufacturer were made at the same time. In this
case we have to know when the form was printed or when the paper was first made that
bears the particular mark.

LESSON 4: THE FOUR TESTS FOR PAPER


1. Preliminary examination
2. Physical test causing no perceptible change.
3. Physical test causing a perceptible change.
4. Chemical test.

1. Preliminary Examination – deals with the appearance of the document and the following
are observed:
a. Folds and creases
b. Odor
c. Impression caused by transmitted light- gives indication of color, translucency where
tampering is made, change in tint which indicates substitution of sheets of paper,
watermarks and wire marks.
d. Presence of discoloration in daylight and under the ultraviolet light.

74
Watermark- if present is one of the most important features in the comparison of paper. It is
distinctive mark or design placed in the paper at the time of its manufacture, by a roll usually covered
with a wire cloth known as dandy roll which serves as means whereby the paper can be identified as
the product of a particular manufacturer.

Wire mark- Marks produced on paper by the flexible wire soldered to the surface of the dandy roll
that carries the watermark.
2. Physical Test Causing No Perceptible Change- a test applied on paper without perceptible
changing or altering the original appearance of the document.
a. Measurement of length and width – to indicate that they originated from the same
manufacturer if two pieces are found to be exactly the same.
b. Measurement of thickness.
c. Measurement of weight or unit area.
d. Color of the paper- it is closely related to its brightness. A side-to-side comparison maybe
made in well-diffused light. Observation of color is influenced by the texture, gloss, finish,
type of illumination and the element of human error.
e. Texture
f. Gloss- gloss and texture maybe determined by visual observation in good daylight or
under different kinds of illumination.
g. Opacity- the quality of paper that does not allow light to pass through or which prevents
dark objects from being seen through the paper.
h. Microscopic Examination/inspection- for possible presence of dirt, foreign particles,
imperfections, wire marks or certain unusual fibers. These maybe a deciding factor in
determining whether or not the same manufacturer made two pieces of paper.

3. Physical Examination Causing a Perceptible Change – this is done only if sufficient


samples are available and if prior authorization from the court is required this to be done.
a. Bursting strength or “pop” test – the apparent pressure necessary to burst a hole in a sheet
when property inserted in a suitable instrument.

b. Folding endurance test- it is obtained on an instrument that registers the number of


alternate folds the paper will stand before breaking.

c. Accelerated aging test- there are some methods of aging a document artificially namely:

1. Soaking in coffee solution


2. Soaking in tea solution
3. Exposure to charcoal
4. Ironing
5. Heating in an oven
6. Exposure to ultraviolet light

d. Absorption test – maybe made to determine either the rate of absorption of the total
absorption of the paper. A strip of paper is suspended in water or ink or other liquid.

4. Chemical Test- this test determines the fiber composition, the loading material and the
sizing material used in the paper.

75
a. Fiber composition- the examination is purely microscopic, and it determines the
material used and the nature of processing. This maybe determined by boiling a small
piece of document in 5% sodium hydroxide. The liquid poured off the fragment of the
paper washed and teased out on a glass slide and stained following the the color
observed under th microscope.

Reagents:
a. 2 grams potassium iodide, 1.5 grams iodine, 2ml glycerine and 20ml water
b. 1) 20 grams zinc chloride ad 10 ml water
2) 2 grams potassium iodide, 1 gram iodine and 5 ml water
Mix 1 and 2 allow the mixture to stand and decant clear supernatant liquid for use ( the
solution is zinc chloroiodine)
c. 1 gram ml phloroglucine, 25 ml water and 5 ml conc HCl
d. 10% solution of aniline sulfate.

b. Sizing material – the sizing materials maybe tested by:

Procedure and Results:

1. Gelatine – is extracted by boiling the paper in water. The solution is tested with
dilute tannic acid solution. Positive result is yellow precipitated.
2. Rosin – this is extracted by heating the paper on water bath with 95% alcohol. The
solution obtained is evaporated to dryness and the residue dissolved in acetic
anhydride , cooled, transferred to a porcelain dish and strong sulfuric acid is added.
Positive result is reddish -violet color that quickly changes to red brown.
Simple test for Rosin- Place a few drops of ether on paper and if rosin is present a
brown ring will be formed when ether evaporates.
3. Starch- add dilute iodine solution on the paper. Blue color is produced if starch is
present.
4. Casein- It can be detected by addition of Millon’s reagent on the paper. Pink color
appears if casein is present.

LESSON 5: THE ANALYSIS FOR INK

Some of the most important questions that arise in the analysis of ink are:
1. Whether the ink is the same with the same or like or different in kind from ink on the other
parts of the same document or on the other documents.
2. Whether two writings made with the same kind of ink were made with identical ink, or inks
of different qualities or in different conditions.
3. Whether an ink is as old as it purports to be.
4. Whether documents of different dates or a succession of differently dated book entries
show natural variations in ink writing or whether the conditions point to one continuous
writing at one time under the same condition.

76
Types of Ink
1. Gallatonic ink or iron-nutgall ink (blue) – today the most frequently used ink for making
entries in record books and for business purposes. Gallatonic ink is made of a solution of
iron salt (ferrous sulfate) and nutgall (iron gallotannate). This ink can penetrate into the
interstices of the fiber and not merely on the surface thus making its removal more difficult
to accomplish. The color changes undergone by this ink in the process of oxidation provides
a valuable means of estimating the approximate age of the writing.

Blue- with the naked eye; very recent


Violet- less recent
Black- still less recent

Changes undergone by the gallotonic ink:


a. First reaching a maximum degree of blackening within the first year or two.
b. Then fades gradually over a period of many years until only a rust colored deposit
remains.
This period of time can be stated only approximate since the oxidation processes are
retarded or accelerated according to the degree of atmospheric humidity, the light, the
quality of the ink itself, the paper, the condition of blotting, condition of storage etc.
2. Logwood ink- the color is dependent on the inorganic salt added. It is made of saturated
solution of logwood to which a very small amount of potassium dichromate is added.
Hydrochloric acid is added to prevent formation of precipitate. Phenol is added as
preservative. The ink does not corrode steel pen. Will not wash off the paper even fresh.
3. Nigrosine ink or aniline ink ( blue black or purple black)- made of coal tar byproduct
called nigrosine dissolved in water. It is easily smudge, affected by moisture and can be
washed off from the paper with little difficulty.
4. Carbon ink or chines ink or india ink- the oldest ink material known. Used to produce
deep black writing ink. Made of carbon in the form of lampblack. Does not penetrate deeply
into the fibers of the paper so that it may easily be washed off.
5. Colored writing ink- today most all colored inks are composed of synthetic aniline
dyestuffs dissolved in water. In certain colored inks, ammonium vanadate is added to
render the writings more permanent.
6. Ballpoint pen ink- made of light fast dye solution in glycol type solvents like glycol or eleic
acid. Paper chromatography can best analyze this ink.

Test for Ink


1. Physical Method/Test- applied to determine the color and presence of alterations,
erasures, destruction of sizes with use of stereoscope, handlens, and microscope.
2. Chemical Test or Spot Test- a simple test wherein different Chemicals or reagent are
applied on the ink strokes and the chemical reactions or characteristics color reactions or
other changes in the ink are observed.

77
The following table shows the chemical reactions of the different types of inks:

Reagent Gallatonic Gallatonic Logwood Nigrosine Carbon


ink with ink without
provisional provisional
color color
5% HCl Blue Disappear Red Maybe No
with slight smudged effect or
yellow color smudge
with
blotter
10% Blue Disappears Violet-red Maybe No
oxalic smudged effect or
acid smudge
with
blotter
Tartaric Blue Disappears Light brown Maybe No
Acid smudged effect or
smudge
with
blotter
2% NaOH Reddish Reddish Brown Runs, dark No
Brown Brown violet at effect or
edges smudge
with
blotter
10% Disappears Disappears Disappears Brown No
NaOCl effect or
smudge
with
blotter
Chlorine Disappears Disappears Disappears Brown No
Water effect or
smudge
with
blotter
KFe(CN) Blue Blue Red No effect No
(a) effect or
smudge
with
blotter
KCNS Red Red - No effect No
(a) effect or
smudge
with
blotter

78
Determination of Approximate Age of Document

1. Age of ink – no definite procedure which can be given for this determination except when
the color is black, because on the observation that within a few hours the color of ink
writings become darker the dye contained therein is influenced by the light of the room,
oxygen of the air, acidity, or alkalinity of the paper.

There are several methods of determining the degree of oxidation of the ink writing and apparently
these methods depend upon:
a. Physical phenomena such as matching the color of the ink writing with standard colors or
with itself over a period of time.
b. Chemical reaction that may reveal some information concerning the length of time the ink
has been on the paper.

2. Age of paper
a. Through watermarks
b. In certain cases from the composition of paper.

What you will do


Activity 1.1

Direction: Secure an original and a fake document. Paste it on a paper explain the differences
between the two documents.

Answer:

79
What you will do

Activity 1.2
Direction: Below is an example of a suicide note that should be investigated. Given the piece of
evidence, how would you analyze the document?

Answer:

80
Module 6: Chemical Aspect of Document Examination Summary
• Document is an original or official written or printed-paper furnishing information or
used as a proof of something else. Is any object that contains handwritten or
typewritten markings whose source or authenticity in doubt.
• The problems encountered in the analysis of paper are the determination of whether
two pieces of paper originated from the same source, determination of the probable
age of paper and determination of the composition of the paper.
• The examination and comparison of paper may determine the age of the paper as
compared to the age of known document, whether the paper is identical with or
different from another paper whose history is known and whether two sheets of
paper of the same manufacturer were made at the same time.

• The four tests for paper are preliminary examination, physical test causing no
perceptible change, physical test causing a perceptible change and chemical test.
• The two tests for ink are the physical method/test which is applied to determine the
color and presence of alterations, erasures, destruction of sizes with use of
stereoscope, handlens, and microscope and the Chemical test or spot test which is
simple test wherein different Chemicals or reagent are applied on the ink strokes and
the chemical reactions or characteristics color reactions or other changes in the ink
are observed.

References
Canete, A. M., & Sangil, M. B. (2014). Laboratory Manual for Forensic Chemistry and Toxicology ( with
Lecture Guide). Philippines: Wiseman's Book Trading.
Kintz, P., Salomone, A., & Vincenti, M. (2015). Hair Analysis in Clinical and Forensic Toxicology. United
Kingdom: Elsevier Inc.
Saferstein, R. (2014). Criminalistics: An Introduction to Forensic Science 11th Edition. Philippines:
Pearson .

Sanico, F. (2015). Forensic Chemistry Worktext. Philippines: Mutya Publishing Inc.

81
MODULE 7:
GLASS, GLASS FRAGMENTS AND
FRACTURE

82
What this module is about

The purpose of this module is to give students a fundamental understanding of forensic


chemistry. We focus on the chemistry relating to specific evidences namely Glass and Glass
Fragments and Fractures. This module demonstrates the application of relevant techniques to
forensic casework.

This module discusses the following topics:

Lesson 1: Composition of glass


Lesson 2: Analysis of glass
Lesson 3: Glass as evidence of crime
Lesson 4: How glass breaks
Lesson 5: Analysis of glass from vehicles
Lesson 6: Analysis of broken windows
Lesson 7: Broken window caused by bullet holes
Lesson 8: Broken window caused by fist or stone

Read this module and see the wonders of forensic chemistry and their contribution to the
world of elucidation of crime.

What you are expected to learn

After going through this module you are expected to:

1. Analyze broken glass or holes.

How to learn from this module

Here’s a simple guide for you in going about the module:

1. Read and follow instructions carefully.


2. Answer the pretest in order to determine how much you know about the lessons in this module.
3. Read each lesson and do activities that are provided for you.
4. Perform all activities diligently to help and guide you in understanding the topic.
5. Take self-tests after each lesson to determine how much you understood the topic.
6. Answer the posttest to measure how much you have gained from the lessons.

83
MODULE 7: GLASS, GLASS FRAGMENTS AND FRACTURES

Intended Learning Outcome

 Analyze broken glass or holes.

Introduction
Glass is important as physical evidence because it breaks, and pieces are scattered at the
crime scene and on the suspect. It is common type of thing caried away evidence in and burglary and
vehicle hit and run cases. The evidence maybe fragments of a headlight leads found at the scene of hit
and run accident, window glass from the scene of robbery, or glass through which a bullet was fired.

LESSON 1 COMPOSITION OF GLASS

What is Glass?

• Fusion of sand (SiO ), soda (Na O) & lime (CaO) that produces a transparent solid when
2 2
cooled.
• Physical properties: hard, elastic, brittle, non-conductor of electricity, density, refractive
index, etc.

What Types of Glass Are There?

The primary uses for glass are in windows, containers, light bulbs, and eyewear.

• Borosilicate Glass (pyrex): 5% borax (Na B O ) is added to resist breaking when heated
2 4 7
or cooled.

• Colored Glass: metal oxides or colloidal iron (Fe) & sulfur (S) are added to change its color.
• Lead glass: Pb increases refractive index & density

LESSON 2: ANALYSIS OF GLASS

Glass type determinations: This type of determination compares a known sample to a glass
fragment to see if the two samples came from the same source. Glass can be made from a variety of
different materials that differ from batch to batch. The presence of the different materials in the glass

84
makes it easier to distinguish one sample from another. Also, the properties of glass can vary
depending upon the temperature the glass is exposed to during manufacturing. Basic properties, such
as color, thickness, and curvature, can also help to identify different samples of glass just by looking
at them. Optical properties, such as refractive index (RI), are defined by various manufacturing
methods. RI is the way light passes through the glass. This can be measured easily even on small
fragments of glass. These properties help to indicate that two samples of glass could be from the same
source.

Direction of force determinations: This method determines which direction a projectile


went through the glass by evaluating radial fractures in the glass fracture’s first concentric ring. The
determination of force direction is a process easily done by a crime scene technician. The purpose of
this determination is to establish which direction the projectile went through the glass. The method
used to establish this is the 4R Rule: Ridge lines on Radial fractures are at Right angles to the Rear.
The first step in this method is to find radial fractures that are within the first concentric
fracture. Radial fractures are like the spokes of a wheel. Concentric fractures connect the radial
fractures in a pattern similar to a spider web. The next step is to figure out which side of the fragment
was facing in and which side was facing out. Contaminants or residues from the inside surface will
feel different than the outside surface and are helpful in determining the sides. Once the technician
finds radial fracture and determines which side of the glass faced where, they must look at the broken
edge of the glass. When a projectile strikes glass, it creates ridges called conchoidal fractures along
the edge that are visible in profile. These conchoidal fractures are nearly parallel to the side in which
force was applied (the direction which the projectile came from). The side of the glass opposite of the
force is the rear of the glass; this is the side of the glass in which the conchoidal fractures lie at right
angles.

Sequence of force determination: An examiner can establish the sequence of the shots by
considering the radial fracture’s termination points. The first shot’s radial fractures will extend
completely while the subsequent shots’ radial fractures will be stopped or cut off as they meet prior
fractures. Glass analysis can be helpful in various ways. Glass fragments at a crime scene should
always be collected and analyzed because several clues can be gathered about the events that
occurred during the crime. Fragments of glass from headlights at a hit-and-run scene can leave clues
about the unknown vehicle. Also, glass fragments can help police determine which direction the first
bullet was fired through the glass. These clues can be gathered through the analysis of even the
smallest fragments of glass.

LESSON 3: GLASS AS EVIDENCE OF CRIME


In the field of forensic chemistry emphasis is placed on.
1.Automobile glass in case of hit and run.
2. Broken windows cause by pressure, blow or bullet in case of robbery.
3. Broken bottles, drinking glasses, spectacles found at the scene of an assault or other crimes of
violence, which would suggest examination of the soles and heels of a suspect for imbedded glass
fragments.

85
LESSON 4: HOW GLASS BREAKS

Each force causes a deformation that may leave a visible mark or fracture the glass. This can
be used to determine the direction and amount of force. Glass acts initially as an elastic surface and
bends away when a force is applied. When the force increases beyond its tensile strength, it cracks.

How Does Glass Break?

86
Radial cracks form first and are propagated in short segments on the side opposite the force.

Concentric cracks come later from continued pressure on the same side as the force applied.

Types of Fractures by Projectiles

Bullets are a projectile force (load) that can pass through glass.

As the bullet’s velocity increases, the central hole becomes smaller, cracking patterns
become simpler, and the exit hole becomes wider than the entrance hole.

Fractures by Projectiles

87
Which side was the bullet fired from?

Exit side is wider than entry side.

Stress lines on the glass edge of radial cracks form a right angle on the reverse side from the force.
Stress lines on the glass edge of concentric cracks form a right angle on the same side as the force.

Which Bullet Hole Was First?

The sequence of impacts can be determined since crack propagation is stopped by earlier cracks.

88
Putting it Back Together Again?

Examiners can fit together two or more pieces of glass that were broken from the same object.
Because glass is amorphous, no two glass objects will break the same way.

89
LESSON 5: ANALYSIS OF GLASS FROM VEHICLES

Hit and run accidents represent a good percentage of crimes. If an automobile or any vehicle
for that matter discovered in which fragments of the lens can be found, a comparison may be done
with the fragments found at the scene of accident employing the methods for analysis of glass.

LESSON 6: ANALYSIS OF BROKEN WINDOWS

Examination of window fragments in robbery cases is important when there is a question of


“as to whether the glass was broken from outside or inside. Since penal laws specifically provides the
mode of entrance before a crime maybe identified as robbery, this particular kind of examination
becomes very important. General procedure to determine whether the glass was broken from outside

90
or inside or to determine the side from which the pane of glass was broken is to collect the pieace
together as much of the glass is possible in order to study the patterns of the cracks and to be able to
orient the pieaces in the original position.

LESSON 7: BROKEN WINDOWS CAUSED BY BULLET HOLES

Generally, it may be said that the hole produced by a bullet of a strong charge have the sharpest edges;
but if a bullet has been fired from a very long distance and has to have a low speed it will break the
pane in the same manner as will a stone.

It is easy to determine the direction from which the shot was fired.
1. On one side of the hole numerous small flakes of glass will be found to have been flown away
giving the hole appearance of a volcano crater. Such appearance indicates that the bullet was
fired from the opposite direction of the hole from which the flakes are missing.
2. If the shot was fired perpendicular to the windowpane the flake marks are evenly distributed
around the hole.
3. If the shot was fired at an angle from the right, the left side will suffer more flaking than the
right.
4. Excessive flaking on the right side of a windowpane would indicate a shot fired at the angle
from the left. (the direction is taken from the person shooting)

LESSON 8: BROKEN WINDOWS CAUSED BY FIST OR STONE

The direction of the blow in case a fist or stone smashed the window is quite difficult but the
principle of radial crack and concentric crack or fracture will apply.

The principle of 3R’s Rule for Radial Crack – states “ stress lines on a radical crack will be at
right angle to the rear side of the glass.”

The principle of RFC Rule for Concentric Crack- states “ stress lines on a concentric crack will
be at right angle to the front side”, that is the side from which the blow came rather than the rear side.

The rule for concentric crack is the reverse of the 3R’s rule provided the concentric cracks can
be examined is near, preferably adjacent to the point of impact.

Procedure: Piece together as many as you can gather of the glass fragements as possible.
Select a triangular piece bounded by two radical cracks and one concentric crack. The triangular piece
must be adjacent to the point of impact, if it is not available select a piece as close as possible to the
point of impact.

91
What you will do
Self-test 2.2

Direction: Analyze the picture carefully. In the figure below, below, which impact occurred first?

Module 7: Glass, Glass Fragments and Fractures Summary


• Glass is important as physical evidence because it breaks, and pieces are scattered at
the crime scene and on the suspect. It is common type of thing caried away evidence
in and burglary and vehicle hit and run cases.
• Glass can be analyzed as to their type, direction of force and sequence of force
determination.
• In the field of forensic chemistry emphasis is placed on.
• 1Automobile glass in case of hit and run.
• 2. Broken windows cause by pressure, blow or bullet in case of robbery.
• 3. Broken bottles, drinking glasses, spectacles found at the scene of an assault or other
crimes of violence, which would suggest examination of the soles and heels of a
suspect for imbedded glass fragments.

• Glass breaks when the force increases beyond its tensile strength, it cracks.

92
• The principle of 3R’s Rule for Radial Crack – states “ stress lines on a radical crack will
be at right angle to the rear side of the glass.”

• The principle of RFC Rule for Concentric Crack- states “ stress lines on a concentric
crack will be at right angle to the front side”, that is the side from which the blow came
rather than the rear side.

• The rule for concentric crack is the reverse of the 3R’s rule provided the concentric
cracks can be examined is near, preferably adjacent to the point of impact.

References
Canete, A. M., & Sangil, M. B. (2014). Laboratory Manual for Forensic Chemistry and Toxicology ( with
Lecture Guide). Philippines: Wiseman's Book Trading.
Kintz, P., Salomone, A., & Vincenti, M. (2015). Hair Analysis in Clinical and Forensic Toxicology. United
Kingdom: Elsevier Inc.
Saferstein, R. (2014). Criminalistics: An Introduction to Forensic Science 11th Edition. Philippines:
Pearson .

Sanico, F. (2015). Forensic Chemistry Worktext. Philippines: Mutya Publishing Inc.

93
MODULE 8:
ARSON

94
What this module is about

The purpose of this module is to give students a fundamental understanding of forensic chemistry.
We focus on the chemistry relating to specific case, arson. This module demonstrates the application
of relevant techniques to forensic casework.

This module discusses the following topics:

Lesson 1: Arson and arson investigation


Lesson2: Origin of Fire
Lesson 3: Motive
Lesson 4: Development/ Identification of Prime Suspect
Lesson 5: Identification of Fire Setter

Read this module and see the wonders of forensic chemistry and their contribution to the world of
elucidation of crime.

What you are expected to learn

After going through this module, you are expected to:

1. Define the basic concepts of fire as a chemical reaction, the major phases of fire, and
the main factors that influence fire spread and fire behavior

How to learn from this module

Here’s a simple guide for you in going about the module:

1. Read and follow instructions carefully.


2. Answer the pretest in order to determine how much you know about the lessons in this module.
3. Read each lesson and do activities that are provided for you.
4. Perform all activities diligently to help and guide you in understanding the topic.
5. Take self-tests after each lesson to determine how much you understood the topic.
6. Answer the posttest to measure how much you have gained from the lesson.

95
MODULE 8: ARSON

Intended Learning Outcome

 Define the basic concepts of fire as a chemical reaction, the major phases of
fire, and the main factors that influence fire spread and fire behavior.

Introduction
Arson continues to be a serious problem on a global scale resulting in high costs in terms of
property loss, injuries, and fatalities. It can be defined as the deliberate and malicious burning of
property having three main elements. First, there has been a burning of property. Second, the burning
is incendiary in origin, and finally, the burning was started with the intent of destroying the property.
The motives for fire-setting behavior have been classified as vandalism, excitement, revenge, crime
concealment, profit, and extremism.
To prove that the crime of arson has been committed, the cause of the fire has to be
determined. Arson is a unique crime in that evidence at the scene can be destroyed; however, a
systematic investigation may yield sufficient evidence to determine the cause. It is therefore
important that every fire scene is treated as a potential arson crime until proof of natural or accidental
cause has been established.

LESSON 1: ARSON AND ARSON INVESTIGATION

THE FIRE TRIANGLE


Four things must be present at the same time in order to produce fire:
1. Enough oxygen to sustain combustion,
2. Enough heat to raise the material to its ignition temperature,
3. Some sort of fuel or combustible material, and
4. The chemical, exothermic reaction that is fire.
Take a look at the following diagram, called the "Fire Triangle”.

96
Oxygen, heat, and fuel are frequently referred to as the "fire triangle." Add in the fourth
element, the chemical reaction, and you have a fire "tetrahedron." The important thing to remember
is take any of these four things away, and you will not have a fire or the fire will be extinguished.
Essentially, fire extinguishers put out fire by taking away one or more elements of the fire
triangle/tetrahedron. Fire safety, at its most basic, is based upon the principle of keeping fuel sources
and ignition sources separate.
DEFINITIONS OF ARSON
1. Under the old common law
Arson is the willful and malicious burning of a house or house of another man.
Present laws of arson have extended the scope of the crime and cover all kinds of buildings and
structures even including personal property. Now the crime of arson includes the burning of one’s
property.
2. Under the revised penal code
Arson is the destruction of property by fire and the extent of liabilities depends on:
a. Kind and character of the building
b. Its location
c. Extent of damage or value
d. Its state of being inhabited or not.
3. According to Webster
Arson is the malicious burning of a building or property or a dwelling house.
4. Arson is a fire set intentionally.
WHAT CONSTITUTE ARSON?
Willfulness- means intentional and implies that the act was done purposely and intentionally.
Intent- is the purpose or design with which the act is done and involves the will.
Motive- is the moving cause that induces the commission of crime. Something that leads or
influences the person to do something.

97
Malice- denotes hatred or ill will or desire or revenge. It is the intent to do injury to another.
A fire can only be considered arson if all accidental and natural cause of fire can be eliminated.
The mere burning of a house does not constitute the body of crime.
To prove the body of crime it is necessary to show:

a. 1st that the building in questioned burned.


b. 2nd it was burned because of the international criminal act of the accused.
c. To continue burning, there must be some burning or charring. A mere smoking,
scorching or discoloration is not sufficient.
BASIC LINES OF INQUIRY IN THE INVESTIGATION OF ARSON
Arson is the easiest crime to commit but the most difficult to detect, tougher to prove than homicide.
At least in the homicide there is weapon, a gun and there is always a good physical evidence. In arson,
physical evidence that normally aid in convincing criminals may have been wholly destroyed by the
fire itself.

THE FOUR BASIC LINES OF INQUIRY IN THE INVESTIGATION OF ARSON


1. Origin of fire
2. Motive
3. Identification of Prime Suspect
4. Identification of fire setter

LESSON 2: ORIGIN OF FIRE

The first step in recognizing the arson is the exclusion of all accidental and natural causes of
fire.

CAUSES OF FIRE
Fire may belong to anyone of the following:

1. Natural cause without human intervention


a. Lightning
b. Explosion
c. Spontaneous combustion
d. Miscellaneous cause, example: damage to electric cables due to earthquake or
storm; breaking of gas pipes, etc.
2. Accidental cause with or without human intervention
a. Faulty wiring
b. Careless handling of inflammables
c. Children playing with match.
d. Careless smokers
e. Careless handling of electric iron, stoves, candles, cigarette butts, mosquito coils.

98
3. Arson or Touch Off fires (a set fire) – when all natural and accidental causes have
investigator to determine if it is in fact a “touch off” fire.

TELL TALE SIGNS


Tell Tale Signs- sign that maybe obvious that the firearm will suspect arson. These are to be
observed to determine if arson.
Tell Tale Signs

a. Burned building- a type of the building may indicate a set fire under some
circumstances. A fire of considerable size at the time the first apparatus arrive at the
scene is suspicious if it is a modern concrete or semi-concrete building.
b. Separate fire- when two or more separate fires broke out within a building the fire
is certainly suspicious.
c. Color of smoke- some fire burn with little or no smoke but there are exception. The
observation of the smoke must be made at the start of the fire since once the fire has
assumed a major proportion, the value of smoke is lost because the smoke will not
indicate the materials used by the arsonists.

1. While smoke appears before the water comes in contact with fire indicates
humid materials burning like burning hay, vegetable materials, phosphorous
(with garlic odor).
2. Biting smoke- irritating the nose and throat and causing lachrymation and
coughing- indicates presence of chlorine.
3. Black or grayish smoke – indicates lack of air but if accompanied by large flame
it indicated petroleum and petroleum products and rubber, tar, coat, and
turpentine.
4. Reddish brown smoke- indicates nitrocellulose, sulfur, sulfuric acid, nitric
acid, hydrochloric acid.
d. Color of fame- the color of the fame is good indication of the intensity of the fire and
the nature of the combustible substance present.
e. Size of fire- The size of fire should be noticed at the time of arrival and at the
subsequent interval thereafter. Rapid extension of the fire is an indicative of the use
of accelerants.
f. Direction of the travel – fire normally sweeps upward; the travel of the fire is
predictable from knowledge of the construction of the building. Flames tend to rise
upward until on meeting obstacles they project horizontally seek other vetical
outlets. The rate of the travel depends primarily on the direction of the wind and on
the ventilating condition like open doors and windows.
g. Intensity- the degree of the heat given off by a fire and the color of its flame
sometimes indicate that some accelerants have been added to the material normally
absent in the building.
h. Location of flame- investigator should not whether there is more than one apparent
point of origin and should try to estimate the approximate location of each.
i. Odors- many accelerants emit characteristics odors especially liquid like turpentine,
alcohol, kerosene, and gasoline.

99
THE POINT OF ORIGIN
In case of arson, it in this area that the physical evidence of criminals designs likely to be
discovered. This may be established by an examination of the witness by inspecting the debris at the
fire scene or by both. The witness to be investigated is the discoverer of the fire and second the person
who turned in the alarm and lastly any other witness that can be found. Inspection of the crime scene
must immediately be made to determine the point of origin and possibly establish arsonist’s
technique.

FIRE SETTING MECHANISM


The technique, devices, and materials employed by the arsonist vary with his mentality and
with his emotional condition immediately prior to the commission of the offense. An arsonist may
use the simplest method in a match and some papers, or he may use an elaborate means to start a
fire. He may use mechanical method or chemical method to start a fire.
Two Methods to Start a Fire
1. Mechanical Method
2. Chemical Method

I. Mechanical Methods to Start a Fire


a. Matches
b. Candles, cigarettes-slow burning initiating arrangement
c. Mechanical devices as clock mechanism, altered equipment, magnifying glass,
animals tied to ignition devices like portable lamp or stove – usually time
delay arrangement.
d. Electrical system/mishap- usually occurs in modern building that are heavily
equipped of electrical wirings to supply fixtures, machines, and heating
purposes.
e. Inflammable gases as illuminating gas, sewer gas.
f. Heating appliances as heaters, sparkers – heaters like flat iron and toaster,
sparkles like electrical switches, doorbells, telephone boxes.
g. Explosives- nitroglycerine, TNT, mercury fulminate, gunpowder. Fire is a
common consequence of explosion. Nitroglycerine is the most employed.
II. Chemical Methods to Start a Fire.
a. Hot water or ice bag used as receptacle for phosphorous and water ignition
device.
b. Metallic sodium ignited by drops of water.
c. Potassium chlorate, sugar, sulfuric acid.
d. Chemical devices as thermit bombs, phosphorous

INCENDIARY MATERIALS
Incendiary materials are materials used to start a fire. They are combustible fuels.

100
1. Arson chemicals- are incendiary materials often used by arsonists as accelerants.
Possess incendiary properties. Example: alcohol, benzene, petroleum ether, gasoline,
kerosene, naphtha, turpentine.
2. Gases as acetylene, butane, CO, ethylene, hydrogen, natural gas, and propane-
these are common gases resulting in fires from explosion. These when mixed with air
possesses excellent ignition properties and when present in an enclosed area can lead
to explosion.
3. Solids as chlorates, perchlorates, chromates, bichromates, nitrates and
permanganates – are typical families or oxidizing agents that give off oxygen on the
decomposition thus aiding in combustion.

MOST COMMON ACCIDENTAL CAUSE OF FIRE


1. Careless disregarding of cigarettes
2. Careless disposition of readily combustible materials
3. Poorly managed of defective heating system
4. Spontaneous combustion
5. Sun rays focused by bubbles in window panes.
6. Explosion from petroleum products, alcohol and other substances
7. Electrical mishaps

LESSON 3: MOTIVE

Although it is not necessary to establish a positive motive, the face that the element of intent
is essential in proving arson suggests the importance of showing a motive. In cases where great
dependence is placed on circumstantial evidence, it is especially important to prove a motive.
FIVE MOTIVES THAT PREDOMINATE IN ARSON CASES
1. Economic gain
2. Concealment of crime
3. Punitive measure
4. Intimidation and economic disabling
5. Pyromania

TYPES OF PERSON WHO SET FIRE

1. Person with a motive


2. Person without a motive
PERSON WITH MOTIVE TO SET FIRE

a. Those with desire to defraud the insurer.


b. Employees or such other persons who gave grievances.
c. Those who try to conceal evidence of other crimes.
d. Those who set fire for purpose of intimidation.

101
e. Pyromania- an uncontrollable impulse toward incendiarism. A term to
describe a condition of mind leading to an act of arson.
Pyromaniac- a type of person who has passion for fire that can be satisfied only by watching flames.
Person who gets pleasure in watching fireman put out the fire.
PERSON WITHOUT A MOTIVE TO SET A FIRE
a. Mental cases
b. Pathological fire-setters
c. Pyros
d. Psychos

LESSON 4: DEVELOPMENT/ IDENFICATION OF PRIME SUSPECT

The third basic line of inquiry pointed toward the identification of the criminal and his
accomplices of any. To accomplish this identification, it is first necessary to develop what are known
as prime suspect among those having motives and opportunity to set the fire or to pinpoint a prime
suspect in pyro fire by studying the fire pattern.
Suspected fire-setters with rational motivation are usually developed by a check of their activities,
questioning and background study. Friends, eyewitness, and others who may possess information
about the fire are interviewed. The owner of the building, the occupants of the premises affected by
the fire and anyone who may profit by the fire are interrogated.
THE FOLLOWING TECHNIQUE MAY SERVE THE INVESTIGATION
1. Search of the fire scene for physical evidence
2. Background study of policyholders, occupants of premises, owner of the building or
other persons having major interest in the fire.
3. Interviews and interrogations of the person who discovered the fire, the one who
turned the first alarm, firemen, eyewitnesses.
4. Surveillance

LESSON 5: IDENTIFICATION OF FIRE SETTER

The final basic inquiry to be pursued by an arson investigator concerns which of the prime
suspects is the fire setter and who were his accomplices if any. This identification results from the
full development of leads, clues and traces. The testimony of persons particularly eyewitness and the
development of expert testimony maybe of value.
GUIDES IN THE INVETIGATION OF FIRE SCENE
The fire scene holds the key to the origin of fire. A careful methodical and through search of the
scene of a suspicious fire is a basic part of the initial investigation. The following points must be
given due attention.

102
1. The scene must be protected so that the evidence is not destroyed or removed by
the careless persons or the guilty party.
2. Mechanics of search
3. Collection and preservation of evidence.
4. Laboratory aids
COLLECTION AND PRESERVATION OF ARSON EVIDENCE
1. Two to three quartz of ash and soot debris must be collected the point of origin of
fire.
2. Specimen should be immediately packed in an airtight container.
a. Can use new clean paint cans with friction lids because they are airtight,
unbreakable.
b. Wide-mouthed glass jars can be used provided they contain airtight lids.
c. Leave an air space in the container above the debris.
3. Plastic polyethylene bags are not suitable for packing.
4. Fluids found in an open bottles or cans must be collected and sealed.
5. Through search of the scene should be undertaken for ignitors.
6. Collect clothing of the suspect/perpetrator and place in a separate airtight
container.
7. Freeze sample containing soil or vegetation.
DETERMINATION OF ARSON CHEMCAL DEBRIS
1. Preliminary test- the debris is reacted with rhodokrit powder for the possible
absence of inflammable substance as gasoline, kerosene, turpentine, alcohol, etc.
2. The debris is steam distilled and distillate collected.
3. The distillate is analyzed in one of the following instruments for confirmation:

a. Gas Chromatography – Mass Spectra (GC-MS)


b. Gas- Liquid-Chromatography (GLC)
c. High Performance Liquid Chromatography (HPLC)

What you will do


Activity 1.1

Direction: Answer the following questions concisely.

1. To initiate and sustain combustion, what are the requirements?


2. Why are arson investigations difficult to investigate?

103
Module 8: Arson and Arson Investigation Summary
o Four things must be present at the same time in order to produce fire:
enough oxygen to sustain combustion, enough heat to raise the material to its
ignition temperature, some sort of fuel or combustible material, and
chemical, exothermic reaction that is fire.

o Arson is a fire set intentionally.

o A fire can only be considered arson if all accidental and natural cause of fire
can be eliminated. The mere burning of a house does not constitute the body
of crime.

o The four basic lines of inquiry in the investigation of arson are the, origin of
fire, motive, identification of prime suspect, identification of fire setter

o There are two methods to start a fire, mechanical method and chemical
method.

o Incendiary materials are materials used to start a fire. They are combustible
fuels.

o The final basic inquiry to be pursued by an arson investigator concerns


which of the prime suspects is the fire setter and who were his accomplices if
any.

References
Canete, A. M., & Sangil, M. B. (2014). Laboratory Manual for Forensic Chemistry and Toxicology ( with
Lecture Guide). Philippines: Wiseman's Book Trading.

Kintz, P., Salomone, A., & Vincenti, M. (2015). Hair Analysis in Clinical and Forensic Toxicology. United
Kingdom: Elsevier Inc.

Saferstein, R. (2014). Criminalistics: An Introduction to Forensic Science 11th Edition. Philippines:


Pearson .
Sanico, F. (2015). Forensic Chemistry Worktext. Philippines: Mutya Publishing Inc.

104
MODULE 9:
DEOXYRIBONUCLEIC ACID (DNA)

105
What this module is about

The purpose of this module is to give students a fundamental understanding of forensic chemistry.
We focus on the chemistry relating to specific case, DNA. This module demonstrates the application
of relevant techniques to forensic casework.

This module discusses the following topics:

Lesson 1: What is DNA


Lesson2: Facts about DNA
Lesson 3: Guidelines for the documentation of DNA Evidence
Lesson 4: How DNA Analysis is done

Read this module and see the wonders of forensic chemistry and their contribution to the world of
elucidation of crime.

What you are expected to learn

After going through this module, you are expected to:

Relate the use of DNA print evidence to the investigation and prosecution of historical crime
case studies.
Describe the rule of evidence concerning DNA print evidence.

How to learn from this module

Here’s a simple guide for you in going about the module:

1. Read and follow instructions carefully.


2. Answer the pretest in order to determine how much you know about the lessons in this module.
3. Read each lesson and do activities that are provided for you.
4. Perform all activities diligently to help and guide you in understanding the topic.
5. Take self-tests after each lesson to determine how much you understood the topic.
6. Answer the posttest to measure how much you have gained from the lesson.

106
MODULE 9: DEOXYRIBONUCLEIC ACID (DNA)

Intended Learning Outcome:


 Relate the use of DNA print evidence to the investigation and prosecution of historical
crime case studies
 Describe the rule of evidence concerning DNA print evidence

LESSON 1: WHAT IS DNA

DNA, or deoxyribonucleic acid, is a molecule that carries genetic instructions for the
development, functioning, growth, and reproduction of all known living organisms and many viruses.
It is a long, double-stranded helical structure that resembles a twisted ladder, often referred to as a
double helix. DNA is made up of smaller units called nucleotides, which are composed of a sugar
molecule (deoxyribose), a phosphate group, and one of four nitrogenous bases: adenine (A), thymine
(T), cytosine (C), and guanine (G). The nucleotides are linked together by covalent bonds between the
sugar of one nucleotide and the phosphate group of the next, forming a sugar-phosphate backbone.
The nitrogenous bases project inward from the backbone and form pairs, with adenine always pairing
with thymine (or uracil in RNA) and cytosine always pairing with guanine. These base pairs are held
together by hydrogen bonds.

The sequence of nucleotides along the DNA molecule encodes the genetic information. Genes,
which are specific segments of DNA, contain the instructions for producing proteins, which are the
building blocks of cells and perform various functions in the body. The order of the nitrogenous bases
in a gene determines the sequence of amino acids in a protein, which ultimately influences an
organism's traits and characteristics.

DNA is located within the cell nucleus of eukaryotic organisms, where it is organized into
structures called chromosomes. Each species has a specific number of chromosomes, and humans
have 46 chromosomes organized into 23 pairs. DNA replication, transcription (the synthesis of RNA
from a DNA template), and translation (the synthesis of proteins based on the information in RNA)
are essential processes that allow cells to duplicate and function properly.

The discovery of the structure of DNA by James Watson and Francis Crick in 1953 marked a
significant milestone in biology, as it provided a framework for understanding how genetic
information is stored, transmitted, and expressed in living organisms. Since then, DNA has been
extensively studied and is a fundamental concept in fields such as genetics, molecular biology, and
biotechnology.

107
LESSON 2: FACTS ABOUT DNA
Structure Discovery: The structure of DNA was elucidated by James Watson and Francis Crick in 1953.
They proposed the double helix model, which won them the Nobel Prize in Physiology or Medicine in
1962.

Code of Life: DNA contains the genetic instructions that determine the characteristics of living
organisms. It carries the hereditary information passed from parents to offspring.

Long Molecules: The length of DNA molecules can vary widely depending on the organism. In humans,
the DNA in each cell's nucleus is approximately 2 meters long when stretched out, but it is tightly
packaged into a compact structure.

Four Nitrogenous Bases: DNA is composed of four nitrogenous bases: adenine (A), thymine (T),
cytosine (C), and guanine (G). Adenine pairs with thymine, and cytosine pairs with guanine through
hydrogen bonds, forming the rungs of the DNA ladder.
Unique DNA Sequence: The order of the nitrogenous bases along the DNA molecule forms a unique
sequence for each individual, except for identical twins. This sequence provides the genetic code that
determines an organism's traits and characteristics.
DNA Replication: Before a cell divides, its DNA must be replicated to ensure that each new cell
receives a complete set of genetic information. DNA replication is a complex process involving
enzymes and occurs with remarkable accuracy.

Non-Coding DNA: Only a small portion of DNA contains genes that code for proteins. The remaining
DNA, often referred to as non-coding DNA, has regulatory functions or functions that are not yet fully
understood.

DNA in Other Organelles: While most DNA is found in the cell nucleus, DNA can also be found in other
organelles, such as mitochondria and chloroplasts. These organelles have their own smaller genomes.

DNA Fingerprinting: The unique DNA sequence in an individual's genome can be used for
identification purposes. DNA fingerprinting, based on the analysis of specific regions of DNA, is used
in forensics, paternity testing, and evolutionary studies.

DNA Modifications: DNA can undergo chemical modifications, such as methylation, which can affect
gene expression and play a role in various biological processes, including development and disease.

LESSON 3: GUIDELINES ON DOCUMENTING DNA EVIDENCE

When it comes to documenting DNA evidence, it is essential to follow established guidelines and best
practices to ensure accuracy, reliability, and the preservation of the evidence. Here are some general
guidelines for the documentation of DNA evidence:
Chain of Custody: Maintain a strict chain of custody for DNA evidence. Document every person who
handles the evidence, including their names, dates, times, and actions taken. This helps establish the
integrity and continuity of the evidence.

108
Proper Packaging: Use appropriate packaging materials to prevent contamination and degradation of
DNA evidence. Items should be individually packaged in paper bags or envelopes to allow for air
circulation and prevent the growth of moisture and mold. Avoid using plastic containers, which can
retain moisture.
Labeling: Clearly label each DNA sample or evidence container with a unique identifier, such as a case
number or barcode, along with other relevant information, including the date, time, location, and
description of the item.
Documentation: Maintain detailed documentation of all DNA evidence-related activities. This
includes documenting the collection method, location, and the individuals involved in the collection
process. Record any observations or potential sources of contamination.

Photography: Take clear and comprehensive photographs of the evidence at the crime scene and
during collection. Capture different angles, close-ups, and overall views to provide a visual record of
the evidence's condition and location.

Description of Samples: Provide a detailed description of the DNA samples, including their nature,
source, and condition. This information helps to establish the relevance and potential significance of
the evidence.
Record Keeping: Establish a comprehensive record-keeping system to track the movement and
handling of DNA evidence. This should include a centralized database or system where all relevant
information can be stored, accessed, and searched easily.
Documentation Standards: Adhere to recognized standards and guidelines for DNA evidence
documentation, such as those provided by forensic science organizations and regulatory bodies. This
ensures that documentation practices align with industry best practices and are accepted in court
proceedings.
Training and Competence: Ensure that personnel involved in DNA evidence collection and
documentation are adequately trained, competent, and up to date with the latest techniques and
guidelines. Regular training and proficiency testing help maintain quality standards.

Legal Considerations: Familiarize yourself with the legal requirements and procedures related to
DNA evidence in your jurisdiction. Follow any specific guidelines provided by law enforcement
agencies, forensic laboratories, or legal authorities.

It's important to note that these guidelines provide a general overview, and specific procedures may
vary depending on the jurisdiction and the type of case. Consulting with forensic experts and
following the protocols established by your organization or laboratory is crucial to ensure accurate
and reliable documentation of DNA evidence.

109
LESSON 4: HOW DNA ANALYSIS IS DONE

DNA analysis, also known as DNA profiling or DNA testing, involves several steps to extract, amplify,
and analyze DNA samples. Here is a general overview of the process:
Sample Collection: DNA samples can be collected from various sources, such as blood, saliva, hair,
skin cells, or other biological materials. The collection method depends on the nature of the sample
and the purpose of the analysis. Proper collection techniques and precautions are followed to prevent
contamination.

DNA Extraction: The DNA is extracted from the collected sample using specialized extraction
techniques. This typically involves breaking open the cells, separating DNA from other cellular
components, and purifying the DNA to remove contaminants. Various methods, such as organic
extraction, magnetic bead-based extraction, or solid-phase extraction, may be employed.

Quantification: The concentration and quality of the extracted DNA are determined through
quantification. This step ensures that sufficient DNA is available for subsequent analysis.
Quantification can be done using spectrophotometry, fluorometry, or other DNA-specific techniques.
DNA Amplification: To obtain a sufficient amount of DNA for analysis, a technique called polymerase
chain reaction (PCR) is used to amplify specific regions of the DNA. PCR involves heating and cooling
cycles that facilitate the replication of targeted DNA sequences. Primers specific to the regions of
interest are used to initiate DNA amplification.
DNA Profiling: The amplified DNA is then analyzed using different profiling methods. One widely used
technique is short tandem repeat (STR) analysis. STRs are specific repetitive sequences in the DNA
that vary in length among individuals. The number of repeats at multiple STR loci is determined using
capillary electrophoresis or other separation techniques.
DNA Database Comparison: The obtained DNA profile can be compared to known DNA profiles in a
DNA database, such as a forensic database or a database of known individuals. This comparison helps
identify potential matches or associations between individuals or samples.
Data Interpretation: The DNA profile is analyzed and interpreted based on established guidelines and
statistical calculations. The interpretation considers the number of repeats at each STR locus and the
frequency of occurrence of those repeats in the general population. Statistical methods, such as
likelihood ratios, are used to assess the strength of the evidence.

Reporting: The results of the DNA analysis are documented in a report. The report includes
information about the analyzed DNA samples, the methods used, the obtained DNA profile, and any
statistical conclusions. The report is typically prepared by forensic scientists and may be used as
evidence in legal proceedings.
It's important to note that the specific techniques and methodologies used for DNA analysis can vary
depending on the purpose of the analysis, the available resources, and the laboratory's protocols.
Advanced techniques, such as next-generation sequencing, may be used for more comprehensive

110
analysis in certain cases. The analysis is carried out by trained professionals in accredited forensic
laboratories following established guidelines and quality assurance practices.

MODULE 9: SUMMARY

DNA, or deoxyribonucleic acid, is a molecule that carries genetic instructions for the
development, functioning, growth, and reproduction of living organisms. It has a double helix
structure composed of nucleotides, including four nitrogenous bases: adenine (A), thymine (T),
cytosine (C), and guanine (G). DNA replication, transcription, and translation are essential processes.
Some interesting facts about DNA include its discovery by Watson and Crick, its role as the
code of life, the unique DNA sequence in each individual, and the presence of non-coding DNA. DNA
evidence should be documented following guidelines such as maintaining chain of custody, proper
packaging, labeling, detailed documentation, photography, and adherence to legal considerations.
DNA analysis involves several steps, including sample collection, DNA extraction,
quantification, PCR amplification, DNA profiling using techniques like STR analysis, database
comparison, data interpretation, and reporting. It is conducted in accredited forensic laboratories by
trained professionals, following established guidelines and quality assurance practices.

References
Canete, A. M., & Sangil, M. B. (2014). Laboratory Manual for Forensic Chemistry and Toxicology ( with
Lecture Guide). Philippines: Wiseman's Book Trading.
Kintz, P., Salomone, A., & Vincenti, M. (2015). Hair Analysis in Clinical and Forensic Toxicology. United
Kingdom: Elsevier Inc.
Saferstein, R. (2014). Criminalistics: An Introduction to Forensic Science 11th Edition. Philippines:
Pearson .

Sanico, F. (2015). Forensic Chemistry Worktext. Philippines: Mutya Publishing Inc.

111
MODULE 10:
FORENSIC TOXICOLOGY

112
What this module is about

The purpose of this module is to give students a fundamental understanding of forensic chemistry.
We focus on the chemistry relating to specific case, DNA. This module demonstrates the application
of relevant techniques to forensic casework.

This module discusses the following topics:

Lesson 1: Toxicology
Lesson2: Classification of Poisons
Lesson 3: Action of Poison
Lesson 4: Kind of dose
Lesson 5: Entrance of Poison
Lesson 6: Elimination of poison
Lesson 7: Specimen of organs to be submitted
Lesson 8: Dangerous Drugs

Read this module and see the wonders of forensic chemistry and their contribution to the world of
elucidation of crime.

What you are expected to learn

After going through this module, you are expected to:

Understood the nature of the toxicological investigations undertaken in forensic laboratories.

How to learn from this module

Here’s a simple guide for you in going about the module:

1. Read and follow instructions carefully.


2. Answer the pretest in order to determine how much you know about the lessons in this module.
3. Read each lesson and do activities that are provided for you.
4. Perform all activities diligently to help and guide you in understanding the topic.
5. Take self-tests after each lesson to determine how much you understood the topic.
6. Answer the posttest to measure how much you have gained from the lesson.

113
LESSON 1: TOXICOLOGY
Toxicology is the scientific discipline that focuses on studying the adverse effects of chemical,
physical, or biological agents on living organisms and the environment. It involves understanding
how these agents interact with biological systems and the mechanisms by which they cause harm.
The primary goal of toxicology is to assess and predict the potential hazards and risks associated with
exposure to various substances. These substances can include drugs, industrial chemicals,
environmental pollutants, pesticides, toxins produced by microorganisms, and even natural
substances like plant toxins.

Toxicologists investigate the toxic effects of substances by examining their absorption,


distribution, metabolism, and excretion within the body. They study how these substances interact
with organs, tissues, cells, and molecules, and the resulting physiological and biochemical changes
that occur. This knowledge helps in understanding the mechanisms of toxicity and developing
strategies to minimize or prevent harm.

Toxicology has various sub-disciplines, including:


1. Descriptive Toxicology: This branch focuses on identifying and characterizing toxic
substances, their sources, and their effects on organisms.
2. Mechanistic Toxicology: It aims to understand the mechanisms by which toxic substances
interact with biological systems, leading to toxicity.

3. Analytical Toxicology: It involves the development and application of analytical techniques to


detect and measure toxic substances in biological samples, such as blood, urine, or tissues.

4. Regulatory Toxicology: This field deals with the application of toxicological knowledge to
develop regulations and guidelines for the safe use of chemicals and substances. It involves
assessing risks, setting exposure limits, and establishing safety standards.
Toxicologists work in a wide range of settings, including research laboratories, government
agencies, pharmaceutical companies, environmental organizations, and forensic science. Their
findings and expertise contribute to various areas such as drug development, environmental
protection, occupational health and safety, risk assessment, and public health policy.

LESSON 2: CLASSIFICATION OF POISONS


Poisons can be classified based on various factors, including their source, mode of action, chemical
nature, and toxicity. Here are some common classifications of poisons:

Chemical Classification:
Inorganic poisons: These are poisons that do not contain carbon in their molecular structure.
Examples include heavy metals like lead, mercury, arsenic, and cyanide.
Organic poisons: These are poisons that contain carbon in their molecular structure. Examples
include drugs, pesticides, herbicides, and industrial chemicals.

114
Mode of Action:

Irritant poisons: These poisons cause irritation or inflammation of the tissues they come into contact
with. Examples include strong acids, alkalis, and certain gases.

Neurotoxic poisons: These poisons affect the nervous system, leading to symptoms such as paralysis,
seizures, or altered mental states. Examples include nerve agents, some pesticides, and certain drugs.

Cardiotoxic poisons: These poisons affect the heart and cardiovascular system, leading to cardiac
arrhythmias, heart failure, or other cardiac-related symptoms. Examples include certain medications
and plant toxins.

Hepatotoxic poisons: These poisons specifically target the liver, causing liver damage or dysfunction.
Examples include alcohol, certain medications, and industrial chemicals.

Nephrotoxic poisons: These poisons target the kidneys, causing kidney damage or impaired kidney
function. Examples include certain medications, heavy metals, and some solvents.

Source of Poison:
Plant poisons: These are poisons derived from plants. Examples include toxins found in certain
mushrooms, poisonous berries, or toxic plant parts.
Animal poisons: These are poisons produced by animals for defense or predation. Examples include
venom from snakes, spiders, or certain marine creatures.

Microbial poisons: These are poisons produced by microorganisms, such as bacteria or fungi.
Examples include toxins produced by Clostridium botulinum (botulinum toxin) or certain molds (e.g.,
aflatoxin).

Lethal Dose:
Fatal poisons: These are highly toxic substances that can cause death even in small amounts.
Examples include certain potent drugs or highly toxic chemicals like cyanide.

Non-lethal poisons: These substances may cause harm or toxicity but are less likely to result in death,
especially at lower doses. Examples include some medications or household chemicals.

It's important to note that the classification of poisons can overlap, as many substances can have
multiple toxic effects or belong to multiple categories. The classification helps toxicologists and
healthcare professionals understand the properties and potential risks associated with different
types of poisons.

LESSON 3: ACTION OF POISONS


The action of poisons refers to the effects they have on living organisms once they are
introduced into the body. The specific actions of poisons can vary depending on their chemical nature,
concentration, route of exposure, and target organs or systems. Here are some general actions of
poisons:

115
Cellular Damage: Many poisons can cause direct damage to cells and tissues. They may disrupt cell
membranes, interfere with cellular metabolism, or disrupt the normal functioning of cellular
organelles. This can lead to cell death, tissue damage, and organ dysfunction.

Enzyme Inhibition: Some poisons can inhibit specific enzymes within the body. Enzymes are essential
for carrying out various biochemical reactions. Inhibition of key enzymes can disrupt normal
physiological processes and lead to toxic effects.

Neurological Effects: Many poisons can affect the central nervous system (CNS) and peripheral
nervous system (PNS). They may interfere with neurotransmitters, block nerve signals, or alter
neuronal function. This can result in symptoms such as dizziness, confusion, seizures, paralysis, or
altered mental states.

Cardiotoxicity: Certain poisons can have toxic effects on the heart and cardiovascular system. They
may disrupt the electrical conduction of the heart, impair heart muscle function, or cause irregular
heart rhythms. This can lead to symptoms like palpitations, chest pain, arrhythmias, or even heart
failure.

Respiratory Effects: Some poisons can affect the respiratory system, leading to respiratory distress,
difficulty breathing, or lung damage. They may cause irritation, inflammation, or compromise the
function of the lungs.

Gastrointestinal Effects: Certain poisons can irritate or damage the gastrointestinal tract. They may
cause nausea, vomiting, diarrhea, abdominal pain, or gastrointestinal bleeding.

Hepatotoxicity: Some poisons specifically target the liver, causing liver damage or dysfunction. They
may disrupt liver cell function, impair detoxification processes, or lead to inflammation and liver cell
death.

Renal Effects: Certain poisons can damage the kidneys or impair kidney function. They may interfere
with blood flow to the kidneys, disrupt the filtration process, or cause inflammation and damage to
renal tissues. This can result in kidney dysfunction or failure.
Allergic or Immunological Reactions: Some poisons can trigger allergic reactions or immune system
responses in susceptible individuals. These reactions can range from mild allergic responses, such as
skin rashes or itching, to severe allergic reactions like anaphylaxis.
It's important to note that the specific actions of poisons can vary widely depending on the
substance involved. Different poisons have different mechanisms of action and can affect various
organs or systems within the body. The severity of the toxic effects can also vary depending on factors
such as the dose, duration of exposure, and individual susceptibility.

116
LESSON 4: KINDS OF DOSE
In toxicology and pharmacology, there are several types of doses used to describe the amount
of a substance administered or received by an organism. Here are some common types of doses:

Acute Dose: The acute dose refers to a single exposure to a substance over a short period, typically
within 24 hours. It represents the immediate or short-term effects of the substance.

Chronic Dose: The chronic dose refers to repeated or prolonged exposure to a substance over an
extended period, such as weeks, months, or even years. It reflects the cumulative effects of the
substance over time.

Lethal Dose (LD50): The lethal dose is the dose of a substance that is lethal to 50% of the population
or test subjects exposed to it. LD50 is commonly used to assess the relative toxicity of substances and
determine their potential for causing death.
Effective Dose (ED50): The effective dose is the dose of a substance that produces a specific desired
effect in 50% of the population or test subjects. It is commonly used in pharmacology to determine
the therapeutic potency of drugs.
Maximum Tolerated Dose (MTD): The maximum tolerated dose is the highest dose of a substance that
can be administered to a subject without causing unacceptable toxicity or adverse effects. It is often
determined in preclinical or clinical studies to establish a safe dosage range.

No-Observed-Adverse-Effect Level (NOAEL): The NOAEL is the highest dose or exposure level of a
substance in a study or experiment that does not cause any observable adverse effects in the test
subjects. It is used to establish safety thresholds and guidelines for exposure.

Lowest-Observed-Adverse-Effect Level (LOAEL): The LOAEL is the lowest dose or exposure level of a
substance in a study or experiment that causes observable adverse effects in the test subjects. It helps
in determining the dose-response relationship and establishing safety thresholds.
Threshold Dose: The threshold dose refers to the minimum dose of a substance required to elicit a
detectable biological response or effect. It represents the point at which the substance starts exerting
its influence on the organism.
These different types of doses are important in toxicology and pharmacology for understanding the
effects, toxicity, and therapeutic potential of substances. They help in assessing risks, establishing safe
exposure limits, and determining appropriate dosages for medical treatments.

LESSON 5: ENTRANCE OF POISON


The entrance or route of entry of a poison refers to the pathway through which a toxic
substance enters the body. The route of entry plays a crucial role in determining the absorption,
distribution, and toxic effects of the poison. Here are some common routes of entry for poisons:

Ingestion: Ingestion is the most common route of entry for poisons. It occurs when a toxic substance
is swallowed and enters the body through the mouth. This can happen by consuming contaminated
food or water, taking toxic medications or chemicals orally, or intentional ingestion of poisonous
substances.

117
Inhalation: Inhalation involves the entry of toxic substances through the respiratory system by
breathing in airborne particles, gases, or vapors. This route of entry is common for pollutants, gases,
volatile chemicals, and environmental toxins. The lungs provide a large surface area for absorption,
allowing rapid entry of substances into the bloodstream.

Injection: Injection refers to the direct introduction of a toxic substance into the body through a
puncture or injection site. It can occur through various means, including intravenous (IV) injection,
intramuscular (IM) injection, subcutaneous injection, or direct injection into tissues or organs. This
route of entry allows for rapid absorption and immediate effects.
Dermal Contact: Dermal contact involves the absorption of toxic substances through the skin. It
occurs when a person comes into direct contact with liquids, solids, or gases containing toxic agents.
Some substances can penetrate the skin easily, while others require factors such as broken skin or
prolonged exposure to enhance absorption.

Mucous Membrane Contact: Mucous membranes, such as those in the eyes, nose, mouth, and genitals,
can provide entry points for toxic substances. Exposure can occur through direct contact with
substances or by inhaling aerosols or splashes that come into contact with these membranes.
Transplacental: This route of entry is specific to pregnant women. Some toxic substances can cross
the placenta and enter the bloodstream of the developing fetus, potentially causing harm or
developmental abnormalities.
Absorption through Other Routes: Some poisons can enter the body through less common routes. For
example, toxic substances can be absorbed through the conjunctiva (the membrane covering the eye),
rectal or vaginal mucosa, or through open wounds or injuries.

Understanding the route of entry is essential in toxicology because it affects the speed and
extent of absorption, distribution, and the potential target organs or systems affected by the poison.
Different routes of entry may result in varying levels of toxicity and different clinical manifestations.

LESSON 6: ELIMINATION OF POISON


The elimination of a poison from the body refers to the process by which the toxic substance
is removed or metabolized and eventually excreted. The body employs various elimination pathways
to rid itself of poisons. Here are some key elimination routes:

Metabolism: Many poisons undergo metabolism, primarily in the liver, to transform them into less
toxic or more easily excreted metabolites. This metabolic process, known as biotransformation,
involves enzymatic reactions that chemically modify the poison. The metabolites are then eliminated
from the body through other elimination routes.

Renal Excretion: The kidneys play a vital role in eliminating water-soluble substances from the body.
Poisons and their metabolites that are water-soluble or have been converted into water-soluble forms
are excreted through the urine. This process involves filtration, reabsorption, and secretion in the
renal tubules.

118
Biliary Excretion: Some poisons and their metabolites are excreted via the bile into the
gastrointestinal tract. From there, they can be eliminated in feces. Biliary excretion is particularly
important for substances that are poorly water-soluble or undergo enterohepatic circulation, where
they are reabsorbed from the intestines back into the bloodstream.
Pulmonary Excretion: Gaseous poisons, volatile substances, and their metabolites can be eliminated
through the lungs during respiration. These substances diffuse across the respiratory membrane and
are excreted in exhaled air.
Sweat and Saliva: Small amounts of certain poisons can be eliminated through sweat and saliva. This
route of elimination is relatively minor compared to other pathways but can contribute to the overall
elimination of certain substances.

It's important to note that the specific elimination pathways for a poison depend on its chemical
properties, including its solubility, molecular size, and metabolic characteristics. Additionally, the rate
of elimination can vary depending on factors such as the dose, individual metabolism, and the
presence of any underlying conditions that may affect organ function.
Elimination of poisons is a complex process, and the time it takes for complete elimination can vary
widely depending on the substance and individual factors. In some cases, elimination can occur
rapidly, while in others, it may take hours, days, or even longer for the body to eliminate the poison
entirely.

LESSON 7: SPECIMEN OF ORGANS TO BE SUBMITTED


In forensic toxicology, when investigating cases involving potential poisoning or exposure to
toxic substances, various organs and biological specimens can be submitted for toxicological analysis.
The choice of specimens depends on factors such as the suspected toxic substance, the nature of the
case, and the availability of the samples. Here are some common specimens submitted for toxicology
analysis:
Blood: Blood is one of the most commonly analyzed specimens in toxicology. It provides a
comprehensive view of the overall drug or toxicant concentrations in the body. Blood samples can be
collected from peripheral veins or, in certain cases, from central sources such as the heart.
Urine: Urine is another valuable specimen for toxicological analysis. It can provide information about
recent or ongoing exposure to toxic substances and their metabolites. Urine samples can be collected
over a specific time period or as a single spot sample.
Tissues: Various tissues can be submitted for toxicology analysis, depending on the suspected toxic
substance and the circumstances of the case. Commonly analyzed tissues include liver, kidney, brain,
lung, muscle, and adipose tissue. These tissues can provide information on the distribution and
accumulation of toxic substances within specific organs.
Gastric Contents: In cases of suspected acute poisoning or recent ingestion of toxic substances, gastric
contents can be collected and analyzed. This can help identify the presence of toxic substances or
drugs that have not yet been absorbed into the bloodstream.

119
Hair: Hair samples can be analyzed for long-term exposure to certain substances, such as drugs or
environmental toxins. Hair analysis can provide information about chronic substance use, drug
history, or environmental exposures over an extended period.

Saliva: Saliva can be collected and analyzed for the presence of drugs or toxic substances. Saliva
testing is particularly useful for detecting recent drug use or exposure.

It's important to note that the choice of specimens may vary depending on the specific case
and the suspected toxic substances. The selection of appropriate specimens should be done in
consultation with forensic toxicologists or medical professionals experienced in toxicology. Proper
collection, storage, and transportation procedures should be followed to ensure the integrity and
reliability of the specimens for analysis.

LESSON 8: DANGEROUS DRUGS


There are several drugs that are considered dangerous due to their potential for abuse,
addiction, and harmful effects on health. Here are some examples of dangerous drugs:

Opioids: Opioids are a class of drugs that include prescription painkillers like oxycodone,
hydrocodone, and fentanyl, as well as illegal drugs like heroin. Opioids can cause respiratory
depression, overdose, and addiction. The opioid crisis has become a significant public health concern
in many countries.

Cocaine: Cocaine is a powerful stimulant drug that can lead to intense euphoria and increased energy.
However, it also carries a high risk of addiction and can cause cardiovascular problems, seizures, and
mental health issues.

Methamphetamine: Methamphetamine, commonly known as meth or crystal meth, is a highly


addictive stimulant drug. It can have severe physical and mental health effects, including damage to
the cardiovascular system, psychosis, and tooth decay (known as "meth mouth").

Synthetic Cathinones (Bath Salts): Synthetic cathinones, often referred to as "bath salts," are synthetic
drugs that mimic the effects of stimulants like amphetamines or MDMA. They can cause
hallucinations, paranoia, violent behavior, and significant physical and psychological harm.
Synthetic Cannabinoids: Synthetic cannabinoids, also known as "synthetic marijuana" or "spice," are
human-made chemicals that are sprayed onto dried plant material and smoked. They can produce
unpredictable and dangerous effects, including severe agitation, psychosis, seizures, and even death.
Benzodiazepines: Benzodiazepines are a class of prescription drugs commonly used for anxiety,
insomnia, and sedation. However, they can be highly addictive, and misuse or abrupt discontinuation
can lead to severe withdrawal symptoms and overdose.

Hallucinogens: Hallucinogenic drugs like LSD (lysergic acid diethylamide) and psilocybin (magic
mushrooms) can induce profound alterations in perception, mood, and thought processes. While they
may not carry the same level of physical harm or addiction potential as some other drugs, they can
still pose risks, including psychological distress and adverse reactions.

120
It's important to note that the classification of drugs as dangerous is based on their potential
for harm and the associated risks. The dangerous nature of these drugs underscores the importance
of education, prevention efforts, and access to treatment and support for individuals struggling with
substance abuse.

MODULE 10: SUMMARY


Toxicology is the scientific study of the harmful effects of chemical, physical, or biological agents on
living organisms and the environment. It involves understanding how these agents interact with
biological systems and developing strategies to minimize or prevent harm.

Poisons can be classified based on factors such as their source (inorganic or organic), mode of action
(irritant, neurotoxic, cardiotoxic, etc.), source (plant, animal, microbial), and lethality or toxicity level.

Poisons can cause cellular damage, enzyme inhibition, neurological effects, cardiotoxicity, respiratory
effects, gastrointestinal effects, hepatotoxicity, renal effects, or allergic/immunological reactions.
Different types of doses used in toxicology and pharmacology include acute dose (single exposure),
chronic dose (repeated or prolonged exposure), lethal dose (LD50), effective dose (ED50), maximum
tolerated dose (MTD), no-observed-adverse-effect level (NOAEL), lowest-observed-adverse-effect
level (LOAEL), and threshold dose.
Poisons can enter the body through various routes, including ingestion (swallowing), inhalation
(breathing in), injection (direct introduction), dermal contact (skin absorption), mucous membrane
contact, transplacental (passing through the placenta to the fetus), and absorption through other
routes like conjunctiva or wounds.

The body eliminates poisons through metabolism (biotransformation), renal excretion (urine),
biliary excretion (bile into the gastrointestinal tract), pulmonary excretion (exhaled air), sweat and
saliva, and other routes depending on the specific toxic substance.

Common specimens for toxicology analysis include blood, urine, tissues (liver, kidney, brain, etc.),
gastric contents, hair, saliva, and others, depending on the suspected toxic substance and the nature
of the case.
Examples of dangerous drugs include opioids, cocaine, methamphetamine, synthetic cathinones
(bath salts), synthetic cannabinoids, benzodiazepines, and hallucinogens. These drugs are associated
with high risks of addiction, harmful health effects, and potential for overdose or adverse reactions.

References
Canete, A. M., & Sangil, M. B. (2014). Laboratory Manual for Forensic Chemistry and Toxicology ( with
Lecture Guide). Philippines: Wiseman's Book Trading.
Kintz, P., Salomone, A., & Vincenti, M. (2015). Hair Analysis in Clinical and Forensic Toxicology. United
Kingdom: Elsevier Inc.

121
Saferstein, R. (2014). Criminalistics: An Introduction to Forensic Science 11th Edition. Philippines:
Pearson .
Sanico, F. (2015). Forensic Chemistry Worktext. Philippines: Mutya Publishing Inc.

What to do after ( Posttest)

I. Multiple Choice

Direction: Encircle the letter(s) of the BEST ANSWER.

1. Hearsay evidence is inadmissible except on the following:


a. res gestae b. dying declaration c. common reputation d. all of these

2. A kind of evidence which seeks to establish a conclusion by inferences from the facts
proven.
a. circumstantial evidence b. indirect evidence c. hearsay evidence d. all of theses

3. It is a proof of allegation.
a. violence b. evidence c. mistakes d. wrongful acts

4. One of the duties of a forensic chemist is to respond to queries of all investigating units
and render opinions.
a. true b. false c. partly false d. partly true

5. Other terms of real evidence


a. authentic evidence b. autoptic evidence c. austetic evidence d. autistic evidence

6. Cells that are produced by the bone marrow and are necessary for proper clotting of blood.
a. erythrocytes b. red blood cells c. leukocytes d. blood platelets

7. A test for blood which determines whether the stains contain blood or another substance.
a. confirmatory test b. blood grouping test c. precipitin test d. preliminary test

8. A preliminary test for blood which gives intense blue color that is produced immediately as
a positive result.
a. Phenolphthalein Test b. Guaiacum Test c. Benzidine Test d. Leucomalachite Green Test

9. The standard test used to determine whether the stain/blood is of human or animal origin.
a. confirmatory test b. blood grouping test c. precipitin test d. preliminary test

10. Discovered the four blood groups namely Group O, Group A, Group B and Group AB.
a. Land Steiner b. Beinsten c. Albert Einstein d. Michael Faraday

122
11. A condition wherein males have no spermatozoa at all in their seminal fluid.

a. Aspermia b. Oligospermia c. hypogonadism d. epididymitis

12. The following are examinations for seminal stains or seminal fluid except:
a. Florence Test b. Barberio’s Test c. Acid Phosphatase Test d. Phenolphthalein Test
13. A whitish fluid of male reproductive tract consisting of spermatozoa suspended in
secretion of accessory glands.
a. Spermatozoa b. Epithelial Cells c. Semen d. Seminal Fluid
14. Where can we find a fresh semen?

a. Vaginal contents of the victim


b. Hair
c. Skin around the genitals
d. Underclothings

15. In the collection, preservation, packing and transit of semen stained specimen, which
statement is incorrect?
a. Seizure of wearing apparel must be done anytime.
b.. In packaging of wearing apparel there should be no friction between the apparel and the stain.
c. Specimen should not be rolled for transit.
d. Specimen must be thoroughly dried before packing.

16. The following are factors that affect the presence or amount of gunpowder residues except:
a.Type and caliber of the ammunition
b. Length of the barrel of the gun
c. Sunlight
d. Wind velocity and direction

17. A test to determine whether a person fired a gun or not with bare hands.
a. paraffin test b. diphenylamine test c. spectroscopic test benzidine test

18. A blackening of area around the bullet hole.


a. Singeing b. Smudging c. Tattooing d. Soothing

19. A black substance that is formed by combustion, rises in fine particles and adheres to the
side of the barrel conveying the smoke.
a. Rust b. Soot c. Nitrate d. Nitrite

20. Combines with excess oxygen in an explosive to achieve oxygen balance, to prevent the
formation of nitrous oxides (toxic fumes), and to lower the heat of the explosion.
a. Bases b. Antacid c. Combustibles d. Absorbents

21. Outer coating of hair that is composed of overlapping scales.


a. medulla b. cuticle c. cortex d. root

123
22. Scale flattened consisting of overlapping scales with narrow margins that resemble puzzle
pieces. Commonly found in human hairs and many animal hairs.
a. spinous scales b. imbricate scales c. coronal scale d. soft scale

23. Fine hair present on the body after birth and before puberty.

a. Lanugos b. Vellus hair c. Terminal hair d. Full grown hair

24. The smallest unit of a textile material that has a length many times greater than its
diameter.

a. yarn b. fabric c. textile d. fiber


25. A convenient way for analysts to compare the colors of fibers through spectral patterns.

a. microspectrophotometer
b. chromatographic separation
c. Infrared spectrophotometry
d. Photochemistry

II. Essay

Direction: Answer the following question on the space provided.

1. In your own view, what is the most important type of evidence? (Ex. Blood, semen, knife etc.).

Answer:

124
125

You might also like